Sie sind auf Seite 1von 100

PARASITOLOGY: REVIEW FOR FINAL EXAM

LECTURE #1: TREMATODES I - SCHISTOSOMIASIS

1. What is Schistosomiasis?
• Schistosomiasis (called bilharziasis in non-English speaking world) is a disease syndrome caused by infection
from one of several species of parasitic trematodes of the genus Schistosoma.
• Disease develops from the host’s immune response to the shistosome eggs >>> granuloma formation >>> fibrosis
(amount of fibrosis correlates to intensity of infection).
• Most granulomas develop at sites of maximal egg accumulation in the intestine and liver (if S. mansoni or S.
japonicum), or in the GU tract (if S. haematobium).
• Granulomas destroy the ova, but result in fibrotic deposition in host tissues. Eggs trapped in tissues release
enzymes and other antigenic substances that sensitize local host lymphocytes >>> secretion of lymphokines >>>
recruits macrophages, lymphocytes, eosinophils and fibroblasts >>> forms compact cellular infiltrate.
• Early acute granulomas are usually composed of eosinophils and neutrophils as well as mononuclear cells.
• Chronic granulomas are primarily macrophages, lymphocytes, fibroblasts, and multinucleated giant cells.
Collagen deposition and fibrosis cause irreversible pathology.
• Schistosomiasis is a major source of M&M for developing countries in Africa, South America, the Caribbean, the
Middle East, and Asia.
• Three types of clinical disease:
1. Swimmer’s Itch: Kabure,
Clamdigger’s itch
2. Katayama Syndrome: acute
schitosomiasis
3. Chronic Schistosomiasis:
a. Intestinal tract
b. Urogential tract

Adult Schistosoma mansoni (male and female). Note the


longer thinner female held within the gynecophoric canal of
the male.

2. What are the common species of Schistosoma?


• Most disease caused by:
1. Schistosoma haematobium: urinary tract
2. Schistosoma mansoni: colon
3. Schistosoma japonicum: colon
• Less prevalent species:
1. Schistosoma mekongi: colon
2. Schistosoma intercalatum: colon

3. What is the life cycle of Schistosomiasis?


• Schistosomes infect particular species of susceptible freshwater snails in endemic areas.
• The infected snails release cercariae, which are fork-tailed free-swimming larvae (1 mm in length), which survive
in freshwater up to 48 hours, during which time they must attach to human skin or to that of another susceptible
host mammal, or they die.
• Cercariae attach to human hosts utilizing oral & ventral suckers.
• They then migrate through intact skin to dermal veins >>> w/in a few days to the pulmonary vasculature. During
this migration, the cercariae metamorphose, shedding tails and outer glycocalyces while developing double-lipid-
bilayer teguments that are highly resistant to host immune responses.
• The organisms (now schistosomula), incorporate host proteins, including major histocompatibility complexes
(MHCs) and blood group antigens, in their integuments. They stop oxidative metabolism and metabolism shifts to
glycolysis.
• The worms then migrate through the pulmonary capillaries to the systemic circulation, which carries them to the
portal veins where they mature.
• In the portal vasculature, adults pair off, with the thin female entering and remaining in the gynecophoric canal of
the stockier 8-mm male worm. Together they migrate along the endothelium, against portal blood flow, to the
Inferior Mesenteric and Superior hemorrhoidal veins (S. mansoni) and Superiror Mesenteric (S. japonicum) or
vesicular veins and veins draining the ureter (S. haematobium) where they begin to produce eggs (4-6 weeks
post-infection, and continuing for life of worm, typically 3-5 years).
• Eggs pass from the lumen of blood vessels into adjacent tissues and many are shed in the feces (S. mansoni and S.
janponicum) or urine (S. haematobium).
• The eggs are highly antigenic and can induce an intense granulomatous response, as they migrate through tissues.
During this time (approximately 10 days) they begin to mature into miracidia.
• Eggs that are not shed successfully may remain in the tissues or be swept back to the portal circulation (from the
mesenteric vessels) or to the pulmonary circulation (from the vesicular vessels via the inferior vena cava [IVC]).
• The free-swimming miracidia that are shed into freshwater survive 2-3 weeks, during which time they must infect
a susceptible snail to complete the life cycle.
• In snails, miracidia drop their ciliated coat and metamorphose into 2 generations of sporocysts, multiply, mature
into free-swimming cercariae, and exit the snail to seek a human host and begin a new cycle.

Eggs are eliminated with feces or urine . Under optimal conditions the eggs hatch and release miracidia , which swim
and penetrate specific snail intermediate hosts . The stages in the snail include 2 generations of sporocysts and the
production of cercariae . Upon release from the snail, the infective cercariae swim, penetrate the skin of the human host
, and shed their forked tail, becoming schistosomulae . The schistosomulae migrate through several tissues and stages
to their residence in the veins ( , ). Adult worms in humans reside in the mesenteric venules in various locations,
which at times seem to be specific for each species . For instance, S. japonicum is more frequently found in the superior
mesenteric veins draining the small intestine , and S. mansoni occurs more often in the superior mesenteric veins
draining the large intestine . However, both species can occupy either location, and they are capable of moving between
sites, so it is not possible to state unequivocally that one species only occurs in one location. S. haematobium most often
occurs in the venous plexus of bladder , but it can also be found in the rectal venules. The females (size 7 to 20 mm;
males slightly smaller) deposit eggs in the small venules of the portal and perivesical systems. The eggs are moved
progressively toward the lumen of the intestine (S. mansoni and S. japonicum) and of the bladder and ureters (S.
haematobium), and are eliminated with feces or urine, respectively . Pathology of S. mansoni and S. japonicum
schistosomiasis includes: Katayama fever, hepatic perisinusoidal egg granulomas, Symmers’ pipe stem periportal fibrosis,
portal hypertension, and occasional embolic egg granulomas in brain or spinal cord. Pathology of S. haematobium
schistosomiasis includes: hematuria, scarring, calcification, squamous cell carcinoma, and occasional embolic egg
granulomas in brain or spinal cord.

Human contact with water is thus necessary for infection by schistosomes. Various animals, such as dogs, cats, rodents,
pigs, hourse and goats, serve as reservoirs for S. japonicum, and dogs for S. mekongi.

Left image: forked-tailed cercaria of Schistosoma mansoni (body 185-230 x 75-110; tail trunk 185-300 x 60-75;fork 90-
130mM). This is the infective larva that emerges from the snail intermediate host. Right image: Miracidium; emerges
from the egg in freshwater. They are ovoid and about 160mM long, possessing cilia on four rows of epidermal plates. They
have four flame cells functioning as the excretory system and their activity serves to indicate the viability of the ovum.

4. What is the pathophysiology of Acute Schistosomiasis?


• Acute schistosomiasis/Katayama Fever is a serum sickness–like illness that develops after several weeks in
some individuals with new schistosomal infections.
• It may correspond to the first cycle of egg deposition and is associated with marked peripheral eosinophilia and
circulating immune complexes.
• It is most common with S. japonicum and S. mansoni infections and is most likely to occur in heavily infected
individuals after primary infection.
• Symptoms usually resolve over several weeks, but the syndrome can be fatal.
• Early treatment with cidal drugs may exacerbate this syndrome and necessitate concomitant glucocorticoid
therapy.
• Mild maculopapular skin lesions may develop in acute infection within hours of exposure to cercariae.
• “Swimmer’s itch,” a pruritic eruption, may occur at the site of penetration of the cercariae, but is rarely seen with
primary exposure. However, with prior sensitization, Schistosomes dermatitis, a pruritic rash, may result.
• The greatest pathological responses occur following avian Schistosomes, probably b/c subQ cercarial death occurs
shortly after they penetrate skin.
• Symptoms: fever, urticaria, melena, marked eosinophilia, diarrhea (bloody), lymphadenopathy, nausea &
vomiting, hepatosplenomegaly, hyperglobulinemia, respiratory symptoms, abdominal pain, interstitial
pneumonitis.

5. What is the pathophysiology of Chronic Schistosomiasis?


• Chronic infection much more common
• Although cercarial and adult worms are minimally immunogenic, schistosomal eggs are highly immunogenic and
induce vigorous circulating and local immune responses. Egg-induced immune responses result in granuloma
formation, and associated fibrotic changes.
• GI Effects: Egg retention and granuloma formation in the bowel wall (usually S mansoni or S japonicum) >>> gut
wall inflammation, microabscesses, and polyposis >>> diarrhea (often bloody) and cramping >>> severe disease
leads to colonic or rectal stenosis.
1. colonic polyposis can present as a protein-losing enteropathy
2. inflammatory masses in the colon may mimic carcinoma
3. patients with heavy bowel wall involvement have an increased rate of recurrent salmonella infection,
generally with positive blood cultures and negative stool cultures.
• Liver Disease: unshed eggs are swept back to the portal circulation, lodge there and induce granulomatous
reactions in the portal tracts. Heavy infestations are more likely to produce hepatic disease. Symmers’ Pipe-
stem fibrosis may occur if heavy infection for years. Although hepatocellular function is spared, periportal
fibrosis can lead to portal hypertension with the usual possible sequelae, including splenomegaly, ascites,
esophageal variceal bleeding, caput medusa, and development of portosystemic collaterals. Co-infection with
hepatitis B or hepatitis C can accelerate hepatic dysfunction and raise the risk for hepatocellular carcinoma.
• Pulmonary Effects: through these collaterals (or directly from the IVC in the case of bladder wall
schistosomiasis), eggs can reach the pulmonary circulation. The resulting pulmonary granulomatosis and fibrosis
can lead to Hypertrophic Osteoarthropathy (clubbing), and pulmonary hypertension and cor pulmonale with
a high mortality rate. In severe cases, patients can develop Right HF.
• GU Effects: egg retention and granuloma formation in the urinary tract (S. haematobium) >>> hematuria (10-12
days post-infection), dysuria, bladder polyps and ulcers, and even obstructive uropathies. Can see sandy patches
on cystoscopy (viable eggs in inflamed bladder mucosa), which are pathognomonic. Later manifestations are
proteinuria (usually in nephritic range), calcifications in bladder, obstruction of ureters, renal colic,
hydropnephrosis, and renal failure. Secondary bacterial infections are common.
1. S. haematobium infection is also associated with an increased rate of squamous cell caricinoma of
the bladder tends to be well differentiated w/ local mets. Typically associated with mutations of
p53 and cyclin-dependent kinase inhibitor 2 tumor suppressor genes.
2. SH causes genital disease in 1/3 of infected women >>> wart-like lesions that can be mistaken for
other diseases, and vulval Schistosomiasis can facilitate the transmission of HIV.
3. SM and HBV >>> higher risk of hepatocellular carcinoma
• CNS Effects: eggs can deposit in cranial or spinal cord vasculature >>> resulting in transverse myelitis (S.
haematobium and S. mansoni), or cerebral disease & seizures (usually S. japonicum). Steroids are important in
treating these infections, because edema and inflammation cause symptoms, also use anticonvulsants.
• General: ectopic egg deposition can lead to additional clinical syndromes including involvement of skin, lung,
brain, muscle, adrenal glands, and eyes.

Schistosoma mansoni in intestinal tissue, acid fast staining of the egg shell is highly characteristic.

6. What is the distribution of Schistosomiasis infections?


• Transmission: exposure to infected fresh water
• infections uncommon in US, b/c neither susceptible snail species nor chronically infected human reservoirs
sufficient to infest freshwater exist.
• However, pathogenic schistosomes can survive and replicate in human hosts for years and even decades.
Therefore, persons who have traveled or immigrated to the US may present to EDs with active cases of acute or
chronic schistosomiasis and/or the associated end-organ complications of this disease.
• Globally, schistosomiasis is a major source of M&M, endemic areas are tropical and subtropical zones
• 250 million people have active schistosomal infection; 120 million people have symptoms, and 20 million are
severely ill.
• Disease tends to be worse in areas with poor sanitation, increased freshwater irrigation usage, and heavy
schistosomal infestation of human and/or snail populations.
1. by major dams, small dams and irrigation projects in developing countries
2. refugees and population shifts
• Schistosomal species vary with geography:
1. S. mansoni and S. haematobium infections predominate in sub-Saharan Africa.
2. S. mansoni is endemic in parts of South America and the Caribbean.
3. S. japonicum is common in China, Indonesia, and the Philippines
4. S. mekogni is common in
Kampuchea, Laos

7. How is Schistosomiasis diagnosed?


• Detection of eggs in feces or urine – may
require 3 specimens
• In patients with typical clinical presentation,
but negative urine and feces specimens, a
bladder biopsy is done.
• Antibody detection with FAST-ELISA is
good for patients not shedding eggs, but
antibodies persist after parasitologic cure.

Schistosoma mansoni egg (115-175 x 45-70mM)


with large lateral spine, and a ciliated miracidium
(larva) is evident within the egg.

8. What is the prognosis for Schistosomiasis infection?


• Although most individuals with chronic schistosomiasis have few or no symptoms, significant morbidity can
develop – acute infection is associated with mortality rates of up to 25%.
• Hepatosplenic disease with portal hypertension is the most common long-term serious outcome, followed by
cardiopulmonary involvement, obstructive nephropathy, bacteremia, and malignancy.
• End-stage hepatosplenic disease with variceal bleeding, pulmonary hypertension with cor pulmonale, and central
nervous system disease are associated with high mortality rates.
• Children and adolescents are infected most often and are infested most heavily.

9. How is Schistosomiasis treated?


• Praziquantal, a pyrazinoisoquinolone derivative, is absorbed well, but undergoes extensive first-pass hepatic
clearance.
• Mechanism: stimulates Ca2+ influx, associated with Ca2+ destruction of tegument in adults
• Side effects: nausea & vomiting
• CMI important
• Drug’s precise actions unknown. It appears to cause tetanic contractions and tegumental vacuoles >>> causing
worms to detach from veins and die.
• Cure rate 60-90%, with substantial decreases in egg and worm burdens in those who aren’t cured.
• Patients who continue to shed viable eggs should be re-treated.
• Treat patients with CNS involvement with steroids and anti-convulsants.

10. What are the different categories of medical parasitology?


• Medical Protozoology
1. parasitic ameba
2. flagellates
3. ciliates
4. apicomplexans (malarial)
5. microsporidia
• Medical Helminthology
1. Nemathelminthes (roundworms): a phylum composed of 3 classes of roundworms. They are more
advanced than flatworms, but less advanced than earthworms. They have thin, round bodies, with no
segments. They live in salt water, fresh water, and soil.
a. hookworm
b. Pinworm
c. trichinella
2. Platyhelminthes (Flatworms): a phylum composed of three classes of flatworms.
They are the simplest of the worms, and live in fresh and salt water.
a. Turbellaria (planarians) is free-living
b. Trematoda (flukes) is parasitic
c. Cestoda (tapeworms) is parasitic
• Medical Entomology
1. arthropods: insects, arachnids, crustacea, centipedes, and milipedes

11. What are the differences between immunity to different strains of Schistosomiasis?
• There is a difference in development of immunity between SM and SJ
• Early: blocking antibodies (IgG2, IgG4, IgM) that block binding of protective IgE and IgG2a, thereby stopping
killing of infectious agents
• Later: IgE and IgG4 compete for binding sites and attack invading larvae >>> humoral immunity of host then has
upper hand
• Eosinophils and macs bind via Fc receptors to specific IgE, IgA and IgG2a on Schistosomium >>> eosinophils
degranulate
1. Major Basic Protein (MBP)
2. Eosinophilic Cationic Protein (ECP)
3. Eosinophil Peroxidase (EPO)
4. Reactive Oxygen Intermediates (ROI)
• C3 also acts with antibodies to enhance response

12. Discuss the involvement of cell mediated immunity in response to Schistosoma infection.
• Early: large volume granulomas form with Eosinophils, PMNs, and monocytes
• Later: modulation occurs with macs, lymphocytes, Giant cells and fibroblasts
• Increase in antibodies acts to trap eggs and egg antigens
• Granulomas is TH2 dominated
• Increased eosinophils levels driven by IL-4, IL-5, IL-13 secretion

13. What are the lab characteristics of Schistosoma?


• Stain with acid fast stain
• Has lateral spine

14. What are some characteristics of Trematodes?


• Trematodes are flukes of the class Trematoda; phylum Platyhelminthes. Trematodes are divided into the
Schistosomes or blood flukes and the tissue flukes.
• Dorsoventrally flattened
• Exulusively endoparasitic
• Organs of attachment:
1. anterior sucker (oral)
2. ventral sucker (acetabulum)
• Complex life cycle: 1-2 intermediate hosts required, w/ first intermediate host always being a snail

LECTURE #2: TREMATODES II – LIVER FLUKES & LUNG FLUKES

1. What are Liver Flukes?


• There are 6 types of liver flukes
1. Clonorchis sinensis
2. Opsithorchis felineous & viverrini
3. Fasciola hepatica
4. Dicrocoelium dendriticum
5. Metorchis conjunctus
2. What is Clonorchiasis?
• An infection of the biliary tract by Clonorchis sinensis, the Chinese liver fluke.
• Distribution: over 20 million infections, endemic in Japan, Korea, China, Taiwan, Vietnam
• Source: ingestion of raw, pickled or smoked fish that have been raised in ponds contaminated with soil from
human or animal feces.
• Complications: secondary bacterial infection and cholangiocarcinoma.

Chlornorchis in bile ducts. Note periductal fibrosis, hyperplasia of biliary epithelium, and bile capillary
proliferation.

3. What is the morphology of Clonorchis sinensis?


• 1-2.5 cm worm that inhabits intrahepatic bile ducts
• Uses suckers to hold on to biliary epithelium
• Adult parasites are elongated, opalescent grey and 10-25mm in length by 3-5mm in width.
• The tegument in the larval fluke is spiny, but in the adult parasite it’s smooth, in contrast to other flukes such
as Fasciola or Paragonimus.
• The gut is bifurcated as with all flukes, but not branched.
• The ventral sucker is smaller than the oral sucker and is situated approximately a quarter of the way down the
parasite from the anterior end.
• The testis are situated at the posterior end of the parasite and are highly branched.
• The eggs are ~30 x 15 microns in size, and the operculum is situated in a rim with distinct shoulders.

Three Clonorchis ova (20-30 x 15-17µm) in stool.

4. What is the life cycle of Clonorchis sinensis?


• Adult worms live in the distal biliary ducts (and sometimes gallbladder or pancreatic duct) of humans
• Eggs are released into bile and pass to feces
• On reaching water, eggs are eaten by a specific species of snail (the first intermediate host)
• Within the snails, miracidia hatch >>> develop into sporocysts >>> rediae >>> cerceriae
• In 4-6 weeks, cercariae are released into the water and penetrate the muscle of susceptible freshwater scaly
fish
• In fish develop into metacercariae, which become infective in 6 weeks.
• Consumption of infected fish is the source of infection for the definitive hosts – humans and other fish-eating
mammals.
• Once ingested, metacercariae excyst in the duodenum or jejunum and migrate through the ampulla of Vater
and common bile duct to the bile ducts >>> where they mature within 4 weeks and begin to produce eggs.

Life Cycle:

Embryonated eggs are discharged in the biliary ducts and in the stool . Eggs are ingested by a suitable snail intermediate
host ; there are more than 100 species of snails that can serve as intermediate hosts. Each egg releases a miracidia ,
which go through several developmental stages (sporocysts , rediae , and cercariae ). The cercariae are released
from the snail and after a short period of free-swimming time in water, they come in contact and penetrate the flesh of
freshwater fish, where they encyst as metacercariae . Infection of humans occurs by ingestion of undercooked, salted,
pickled, or smoked freshwater fish . After ingestion, the metacercariae excyst in the duodenum and ascend the biliary
tract through the ampulla of Vater . Maturation takes approximately 1 month. The adult flukes (measuring 10 to 25 mm
by 3 to 5 mm) reside in small and medium sized biliary ducts. In addition to humans, carnivorous animals can serve as
reservoir hosts. (www.cdc.gov)
Clonorchis sinensis ova in uterus of worm.

5. What are the clinical manifestations of Clonorchiasis?


• Commonly infected early in life and asymptomatic.
• Benign infections are usually identified by routine stool examination revealing eggs.
• Acute Clonorchiasis: Katayama-like, malaise, fever, WBCs and Eosinophils elevated.
• Patients don’t become symptomatic until at least the 3rd decade of infection >>> the following symptoms:
1. dull pain in upper right quadrant – can be intermittent and last for months to years
2. lassitude
3. anorexia
4. loose stools
5. poor appetite
6. low-grade fever
7. hepatosplenomegaly
8. intermittent fever
• in advanced stages of disease, liver is enlarged, gallbladder is palpable and functioning poorly, some patients
have intermittent fever and jaundice from relapsing cholangitis or pyogenic cholangitis.
• Cholangiocarcinoma is observed in association with clonorchis and deterioration is rapid with a fatal
outcome.

6. What are some potential complications of Clonorchiasis?


• Intrahepatic calculi
• Biliary obstruction due to fibrosis or cirrhosis and goblet cell metaplasia
• Recurrent cholangitis with pyogenic abscesses, endophlebitis, and acute
pancreatitis
• Cholangiocarcinoma

7. How is Clonorchiasis diagnosed?


• By finding Clonorchis eggs in the stool or biliary fluid.
• Endoscopic retrograde cholangiopancreatography (ERCP)
• Transhepatic cholangiogram

Image: Clonorchis sinensis egg (20-30 x 15-17µm), note the operculum or lid.
This structure opens to permit the miracidium to escape.

8. How should Clonorchiasis be treated?


• Praziquantel
• Cure rates of 85-100%

9. How can Clonorchiasis be prevented?


• Through cooking and freezing freshwater fish, or through use of salt in 10% brine solution.
• Educational efforts directed towards changing eating habits – drying, pickling and smoking are ineffective at
killing organisms.

10. What is Paragonimiasis?


• Paragonimiasis is infection by lung flukes of the genus Paragonimus.
• Most common species is the Oriental Lung Fluke, Paragonimus westermani.
• Paragonimiasis is a zoonotic lung infection of wild felines and canines throughout the world, but main
endemic foci of human infections is in Asia, Africa, and Central & South America.
• 20 million infections worldwide – Asia, Africa, Peru, Venezuela, US, Costa Rica
• Adult worms usually live encapsulated in pockets in the lungs, but may be found in extrapulmonary sites.
• Transmission: disease is acquired by ingesting raw crayfish & crab. Contributing factors:
1. large number of reservoir hosts (humans and animals)
2. abundance of first and second intermediate hosts
3. social customs among some Asian populations of eating raw crab, crayfish or shrimp

11. What is the morphology of the Oriental Lung Fluke?


• Adult parasites are reddish brown, ovoid or coffee bean shaped, with broad anterior end
• Their tegument is covered with cuticular spines
• They are bilaterally symmetrical and dorso-ventrally flattened, with anterior oral and ventral suckers
• All are hermaphroditic
• The eggs are golden brown, asymmetrically ovoid with a thick shell, and have a flattened operculum

Left image: Paragonimus westermani adult lung fluke. The


living adult is reddish brown, ovoid, "coffee-bean" in appearance. It is 7-16mm in length x 4-8mm in width.
Right image: Paragonimus westermani golden brown ova (80-118µm x48-60µm).

12. What is the life cycle of the Oriental Lung Fluke?


• Adult flukes live in and lay eggs within a pulmonary cyst.
• The eggs are passed along a tunnel that connects to a large bronchus and are expectorated with sputum or
swallowed and passed through feces.
• On reaching water, the eggs hatch in 3 weeks to form mircidia >>> which invade tissues of snails (first
intermediate host).
• In snails, they undergo asexual transformation into sporocysts >>> rediae
• Cerceriae emerge from snails and penetrate crustaceans (crabs & crayfish are 2nd intermediate host) >>>
become infective-stage metacercariae in 6-8 weeks. Crabs can also acquire infection by eating infected
snails.
• When infected crustaceans are consumed by mammals (definitive host), the metacercariae excyst in the
duodenum >>> penetrate the intestinal wall >>> migrate through abdominal wall >>> grow into flukes in 1
week >>> flukes penetrate the diaphragm and enter lung tissue >>> form pseudocapsule and grow into adult
flukes in 5-6 weeks.
• Then they produce eggs that appear in sputum or feces in 8-10 weeks.
• Flukes can live up to 20 years in humans.
The eggs are excreted unembryonated in the sputum, or alternately they are swallowed and
passed with stool . In the external environment, the eggs become embryonated , and
miracidia hatch and seek the first intermediate host, a snail, and penetrate its soft tissues .
Miracidia go through several developmental stages inside the snail : sporocysts , rediae ,
with the latter giving rise to many cercariae , which emerge from the snail. The cercariae
invade the second intermediate host, a crustacean such as a crab or crayfish, where they encyst
and become metacercariae. This is the infective stage for the mammalian host . Human
infection with P. westermani occurs by eating inadequately cooked or pickled crab or crayfish that
harbor metacercariae of the parasite . The metacercariae excyst in the duodenum ,
penetrate through the intestinal wall into the peritoneal cavity, then through the abdominal wall
and diaphragm into the lungs, where they become encapsulated and develop into adults (7.5 to
12 mm by 4 to 6 mm). The worms can also reach other organs and tissues, such as the brain
and striated muscles, respectively. However, when this takes place completion of the life cycles
is not achieved, because the eggs laid cannot exit these sites. Time from infection to oviposition
is 65 to 90 days.
Infections may persist for 20 years in humans. Animals such as pigs, dogs, and a variety of feline
species can also harbor P. westermani.

13. What is the pathogenesis of infection with Lung Flukes?


• Migrating larval flukes in the lungs and frequently in other tissues induce local necrosis, hemorrhage and
inflammatory exudates >>> followed by fibrous encapsulation into a worm cyst.
• Pathology includes:
1. Lung: granumolatous reactions >>> fibrosis and bronchiectasis, or bronchopneumonia, interstitial
pneumonia, bronchitis, atelectasis, pleural thickening, pleural effusion, angiitis obliterans,
periphlebitis.
2. Abdominal cavity: abscesses and ulcers in muscle, eggs cause pseudotubercle formation in liver,
lymph nodes, gut wall.
3. CNS: abscesses in brain and spinal cord.

14. What are the clinical manifestations of Paragonimiasis?


• In light infections, most patients are asymptomatic.
• In heavy infections, patients may have few symptoms and appear well in spite of severe pathology.
• Acute Stage: corresponds to period of migration and invasion of young flukes. Incubation period of 2-15
days. Often passes undiagnosed, but may include diarrhea, abdominal pain, urticaria, and eosinophilia,
followed by fever, chest pain, cough, dyspnea, malaise and night sweats, and last for a few weeks.
• Pulmonary Paragonimiasis: incubation period of 6 months, predominant symptom cough.
1. Cough, sputum and chest discomfort increase gradually
(with clinical presentation similar to chronic bronchitis)
2. hemoptysis
3. SOB with exertion
4. wheezing
5. mild fever
6. normal hemoglobin with relative eosinophilic
leukocytosis
7. Chest x-ray: ill-defined opacities early on, and later cysts
(ring shadow w/ a crescent-shaped opacity along one
side), extensive infiltration and pleural involvement

Image: Infiltrates due to Paragonimiasis

• Ectopic sites: brain & spinal cord, intestinal wall, lymph nodes,
and subQ
• Cerebral & Spinal Paragonimiasis: some mature flukes migrate
from the lungs through the juguluar or carotid foramen to the
temporal or occipital lobes of the brain >>> necrosis and
eosinophilic granulomatous reactions. Most common in children and males. Patients can die if left untreated,
if not, remission occurs in 1-2 months, but can recur. Patients present with:
1. headache
2. vomiting
3. fever
4. visual disturbances
5. rarely, transverse myelitis
6. spinal tap shows eosinophilic pleocytosis
7. CT shows cysts

15. How is Paragonimiasis diagnosed?


• Eggs in sputum, stool, pleural effusions or cerebrospinal fluid
• Adult flukes in subQ nodules or other surgical specimens
• Chest x-ray with characteristic ring lesions
• Serologic testing with ELISA is useful for ectopic sites

16. How is Paragonimiasis treated, and how can it be prevented?


• Praziquantel cure rates of 100%
• High doses in cerebral disease
• Steroids may be necessary
• Prevention: cook raw crustaceans and use good hygiene

14. What is Fascioliasis?


• Sheep Liver Fluke Disease is an infection caused by Fasciola hepatica, that infects the bile ducts and
gallbladder and extra-hepatic sites.
• Distribution: sheep-raising areas, 2 million infections
• Transmission: humans become infected when they eat vegetation encysted with metacercariae (such as
watercress sandwiches).

15. What is the morphology of Fasciola hepatica?


• F. hepatica is relatively flat, leaf-like and fleshy
• 20-30 mm x 8-13 mm
• Its broader anterior portion is covered with scale-like spines.
• A distinct cephalic cone gives a characteristic shouldered appearance.
• The eggs are light yellowish-brown, large and ovoid with an inconspicuous operculum, measuring 130-150
micrometers x 60-90 micrometers.

Left image: stained Fasciola hepatica. Right image: unstained specimen of F. hepatica, a moderately fleshy fluke
up to 30mm in length by 13mm wide.

Fasciola showing prominent integumental spines.

16. What is the life cycle of Fasciola hepatica?


• Adult parasites live in the large biliary ducts of host. Most important definitive hosts are sheep, but cattle,
horses, camels, pigs, rabbits and deer are commonly infected.
• Eggs pass from host feces and hatch in water.
• Miracidia penetrate various lymnaeid snails (first intermediate hosts) and develop in 4-7 weeks from
sporocysts >>> rediae >>> daughter rediae >>> cercariae
• Mature cercariae emerge from the snail and encyst on various kinds of aquatic vegetation to become
metacercariae
• Metacercariae are ingested, excyst on duodenum >>> migrate through intestinal wall into peritoneal cavity
>>> pass through Glisson’s capsule >>> traverse the liver parenchyma >>> bile duct where they mature. This
takes 3-4 months.
• Life span is estimated to be 9-13 years.

Life Cycle:

Immature eggs are discharged in the biliary ducts and in the stool . Eggs become embryonated in water , eggs release
miracidia , which invade a suitable snail intermediate host , including many species of the genus Lymnae. In the snail
the parasites undergo several developmental stages (sporocysts , rediae , and cercariae ). The cercariae are released
from the snail and encyst as metacercariae on aquatic vegetation or other surfaces. Mammals acquire the infection by
eating vegetation containing metacercariae. Humans can become infected by ingesting metacercariae-containing freshwater
plants, especially watercress . After ingestion, the metacercariae excyst in the duodenum and migrate through the
intestinal wall, the peritoneal cavity, and the liver parenchyma into the biliary ducts, where they develop into adults . In
humans, maturation from metacercariae into adult flukes takes
approximately 3 to 4 months. The adult flukes (Fasciola
hepatica: up to 30 mm by 13 mm; F. gigantica: up to 75 mm)
reside in the large biliary ducts of the mammalian host.
Fasciola hepatica infect various animal species, mostly
herbivores. (www.cdc.gov)

17. What is the pathogenesis of Fascioliasis?


• Migrating metacercariae cause local hepatic parenchymal destruction, necrosis and abscess formation.
• Once they reach the bile ducts and mature, adult flukes may cause epithelial hyperplasia and periductal
fibrosis.
• Adults in the biliary tree may lead to cholangitis and stone formation.
• The gall bladder may commonly be involved and rarely cirrhosis has been reported.
• Ectopic fascioliasis commonly involves lungs, and sometimes subQ tissues, brain and orbits.
Liver fluke (Fasciola) in liver at autopsy.

18. What are the clinical manifestations


of Fascioliasis?
A. Acute Phase
• Patients may develop anorexia,
nausea, vomiting, fever and right
upper quadrant pain
• Hepatomegaly may occur with
tenderness and marked eosinophilia
• Severe illness with prostration and
jaundice is unusual.
• This phase is due to the larval
flukes migrating through the liver
and stopping when they penetrate
the bile ducts.
• Patients may be asymptomtic
during this phase.
B. Chronic Phase
• Most patients have few or no
symptoms after the flukes lodge in the biliary passages.
• Some have epigastric and right hypochondrial pain, diarrhea, nausea, vomiting, hepatosplenomegaly, and
jaundice.
• If the flukes lodge in extrahepatic biliary ducts, may have symptoms of cholelithiasis.
• Abnormal liver function and eosinophilia are common.
• Complications include biliary obstruction.

Fasciola ova in uterus of adult fluke.

19. What is the distribution of Fasciola hepatica?


• Fascioliasis occurs worldwide.
• Human infections with F. hepatica are found in areas where sheep and cattle are raised, and where humans
consume raw watercress, including Europe, Cuba, the Middle East, and Asia, and also in South America.
• Infections with F. gigantica have been reported, more rarely, in Asia, Africa, and Hawaii.

20. How do you diagnose Fascioliasis?


• Eggs in the stool
• Serology tests are also available
Image: Fasciola hepatica egg (130-150µm x 63-90µm). The operculum can be seen in the "7 o'clock" position.

21. How do you treat Fascioliasis?


• Triclabendazole

LECTURE #3: CESTODES I – D. LATUM, TAENIASIS

1. What are some significant characteristics of Tapeworms/Cestodes?


• Infections with tapeworms are one of the oldest recognized afflictions of humankind.
• Cestodes are dorso-ventrally flattened, and divided into sexually complete segments.
• Adult tapeworms posess a scolex or head that may be modified with structures or organelles that serve as
organs of attachment to the small intestine mucosa.
• The bothria are sucking grooves that attach to the ileum or jejunum. They use the bothria to transfer nutrients
through microvilli-like structures from host to worm, so glucose is taken directly from host’s intestinal lumen.
• The digestion portion of the scolex is termed the neck, and it is an area of intense metabolic activity. It is also
the area where new segments, or proglottids, proliferate and form the strobila.
• Cinically, the scolex is important because therapy is aimed at its destruction or elimination.
• The most proximal proglottids are usually immature, while the more distant proglottids are more mature with
sex organs.
• The surface epithelium, or tegument, is actually a syncitium composed of anucleate ectocytoplasm that covers
the body of the tapeworm and an underlying layer containing the nucleated cell bodies or perikarya.
• They have circular and longitudinal muscles, a primitive nervous system, highly complex repro organs, and
primitive excretory system, and complex life cycles.
• They inhibit host enzymes, block host cell motility and limit lymphocyte mitogenic response.
• Infections with adult oranisms typically not life-threatening, but larval infections can be.

2. What are Diphyllobothrium latum?


• Diphyllobothriasis is an infection of the
intestinal tract by the Broad or Fish
Tapeworm.
• Disease is found worldwide.
• Transmission: humans become infected by
ingesting D. latum pleroceroid larvae in fish
or fish eggs that are raw or incompletely
cooked. Varieties of freshwater as well as
some marine fishes are sources of infection.
• Intermediate hosts: crustaceans & fish
• Definitive hosts: humans and fish-eating
carnivores such as bear, cats, dogs.

Image: Diphyllobothrium latum scolex and


immature proglottids. The scolex has a pair of grooves, termed bothria, that are used as adherence structures.

3. What is the morphology of Diphylobothrium latum?


• D. latum ranges in length from 1-12 meters
• Proglottids are usually wider than they are long, and in the center of mature ones there is a characteristic
dark rosette – egg filled uterus. The largest worms have thousands of proglottids.
• The ovoid eggs measure 42-50 microns by 75 microns and have a lid-like operculum at one end and a small
knob at the other end.

Left image: Diphyllobothrium latum egg (42-50 x 59-


75µm). The egg has an operculum which opens in
freshwater to allow egress of the ciliated larva or
coracidium.
R
i
g
ht image: Diphyllobothrium latum gravid proglottid showing the uterus and the muscular uterine pore from which
eggs are discharged into the fecal stream.

4. What is the distribution of D. latum?


• Common in lake country in the US, central Florida, Adirondacks
• Japan
• Argentina
• Africa
• Baltic States
• Israel
• Finland

5. What is the lifecycle of Diphylobothrium latum?


• Feces with eggs from infected hosts are discharged into fresh or brackish water that contains susceptible
crustaceans or fish.
• After the egg hatches, the motile hexacant embryos or coracidia are released into water and must find a host
quickly, if successful, they are ingested by minute crustecea called water fleas >>> which are ingested by
small fish.
• The first stage larva (procercoid larvae) develop in the fish >>> which are then eaten by larger fish >>>
develops into a plerocercoid or sparganum larvae, which is infective for final host.
• Infection occurs when the infected fish are then eaten raw by definitive hosts.

Life Cycle:
Immature eggs are passed in feces . Under appropriate conditions, the eggs mature (approximately 18 to 20 days) and
yield oncospheres which develop into a coracidia . After ingestion by a suitable freshwater crustacean (the copepod first
intermediate host) the coracidia develop into procercoid larvae . Following ingestion of the copepod by a suitable
second intermediate host, typically minnows and other small freshwater fish, the procercoid larvae are released from the
crustacean and migrate into the fish flesh where they develop into a plerocercoid larvae (sparganum) . The plerocercoid
larvae are the infective stage for humans. Because humans do not generally eat undercooked minnows and similar small
freshwater fish, these do not represent an important source of infection. Nevertheless, these small second intermediate
hosts can be eaten by larger predator species, e.g., trout, perch, walleyed pike . In this case, the sparganum can migrate
to the musculature of the larger predator fish and humans can acquire the disease by eating these later intermediate infected
host fish raw or undercooked . After ingestion of the infected fish, the plerocercoid develop into immature adults and
then into mature adult tapeworms which will reside in the small intestine. The adults of D. latum attach to the intestinal
mucosa by means of the two bilateral groves (bothria) of their scolex . The adults can reach more than 10 m in length,
with more than 3,000 proglottids. Immature eggs are discharged from the proglottids (up to 1,000,000 eggs per day per
worm) and are passed in the feces . Eggs appear in the feces 5 to 6 weeks after infection. In addition to humans,
many other mammals can also serve as definitive hosts for D. latum.

6. What are the clinical manifestations of infection with Diphyllobothrium latum?


• Most patients are asymptomatic or mildly symptomatic.
• Small percentage develop megaloblastic anemia.
• Worm is not invasive, and there is little pathologic damage associated with infection other than that related to
competition between the host and parasite for absorption of vitamin B12.
• Symptoms may include:
1. weight loss
2. increased or decreased appetite
3. symptoms of pernicious anemia (weakness, fatigue, peripheral neuropathy, diarrhea, weight loss,
decreased appetite, etc.)
4. massive worm can obstruct small intestine >>> intersussception >>> compromising intestinal
circulation.
• Symptoms in children: worms secrete proteins into intestinal lumen that can lead to syncope or seizures.
1. abdominal pain
2. changes in appetite
3. seizures
4. syncopal attacks

7. Discuss the pernicious anemia-like syndrome that can be associated with D. latum infection?
• D. latum has a high affinity for Vitamin B12 >>> producing a hyperchromatic macrocytic megaloblastic
anemia without thrombocytopenia, that resembles pernicious anemia both clinically and hematologically.
• The anemia is usually moderate, but can be severe, and is characterized by pallor, glossitis, dyspnea,
paresthesia, disturbance of movement and coordination, and impairment of deep senses, subacute combined
degeneration >>> decreased hemoglobin and decreased oxygen in blood.
• Vitamin B12 absorption based on Intrinsic Factor
• binds to ileal receptor (Ca2+)
• pancreatic factor also increases absorption
• 40% of infected have low B12, 2% develop anemia, worm uptake of B12 is 1000:1

8. How is an infection with Diphyllobothrium latum diagnosed?


• Through characteristic eggs with operculum.

9. How is an infection with Diphyllobothrium latum treated?


• Praziquantel or Niclosamide (not available in US)
• Affects voltage-gated Ca2+ channels
• A single dose is curative in >90% of patients

10. How do you prevent infection with Diphyllobothrium latum?


• Careful cooking or freezing of fish
• Drying or pickling in brine

11. What is Taeniasis?


• The pork tapeworm is Taenia solium.
• The beef tapeworm is Taenia saginata.
• They are both common human parasites, and infection occurs in people
who have eaten raw or improperly cooked pork or beef.
• Humans are the only definitive hosts.
• T. saginata is a cosmopolitan parasite, with regions of high endemnicity
in Latin America , Africa, the Middle East, and Central Asia.

12. What is the morphology of Taenia solia and Taenia saginatum?


13. A. T. saginata
• Adults 5-12 meters in length
• 1000-2000 proglottids
• The scolex has 4 sucking disks and lacks a crown or rostellum with
hooks (unarmed scolex).
B. T. solium
• 1.5-8 meters in length
• 1000 proglottids
• Scolex with a well-developed crown of rostellar hooks (armed scolex).

Top image: T. solium with armed scolex. Bottom image: T. saginita with unarmed scolex

14. What are the life cycles of Taenia solia and Taenia saginatum?
• Cattle or pigs become infected by ingesting eggs while grazing in low-lying pastures or barnyards that are
contaminated with human feces >>> eggs hatching inside the animal >>> embryos then penetrate the
intestinal mucosa >>> enter animal’s circulation and are transported throughout the body >>> embryos encyst
in animal’s striated muscle >>> develop into larvae called cysticercus cellulosae (in pigs) or cystericus bovis
(in cattle), which are infectious if ingested by definitive host.
• Upon consumption of infected meet that is improperly cooked, the cystericus is activated by gastric juices and
the scolex evaginates and attaches to a person’s jejunal wall >>> becomes a mature tapeworm.

Life Cycle:

Life cycle of Taenia saginata and Taenia solium


Humans are the only definitive hosts for Taenia saginata and Taenia solium. Eggs or gravid proglottids are passed with
feces ; the eggs can survive for days to months in the environment. Cattle (T. saginata) and pigs (T. solium) become
infected by ingesting vegetation contaminated with eggs or gravid proglottids . In the animal's intestine, the oncospheres
hatch , invade the intestinal wall, and migrate to the striated muscles, where they develop into cysticerci. A cysticercus
can survive for several years in the animal. Humans become infected by ingesting raw or undercooked infected meat .
In the human intestine, the cysticercus develops over 2 months into an adult tapeworm, which can survive for years. The
adult tapeworms attach to the small intestine by their scolex and reside in the small intestine . Length of adult worms
is usually 5 m or less for T. saginata (however it may reach up to 25 m) and 2 to 7 m for T. solium. The adults produce
proglottids which mature, become gravid, detach from the tapeworm, and migrate to the anus or are passed in the stool
(approximately 6 per day). T. saginata adults usually have 1,000 to 2,000 proglottids, while T. solium adults have an
average of 1,000 proglottids. The eggs contained in the gravid proglottids are released after the proglottids are passed with
the feces. T. saginata may produce up to 100,000 and T. solium may produce 50,000 eggs per proglottid respectively.

15. What is the distribution of Taenia solia and Taenia saginata?


• Both species are worldwide in distribution.
• Taenia solium is more prevalent in poorer communities where humans live in close contact with pigs and eat
undercooked pork, and in very rare in Muslim countries.
16. What are the clinical manifestations of Taeniasis?
• Most tapeworm carriers are asymptomatic and only become aware of infection when they see proglottids in
feces or notice strange sensations due to spontaneous movement of tapeworm segments through the anus.
• Mild GI symptoms can occur in some patients such as nausea and vague epigastric pain
• Also anorexia, headaches, dizziness, diarrhea
• Cysticercosis infection with the larval form of Taenia solium is a potentially serious complication of T. solium
infections.

17. How do you diagnose Taeniasis?


• Infection often recognized due to elimination in feces
• The standard method for differentiating between species is to count the number of primary uterine branches in
gravid proglottids
1. T. solia has ≤10
2. T. saginata has ≥12
• Visual demonstration of eggs in stool is not specific, b/c solia and saginata eggs appear identical under light
microscope.

18. How do you treat Taeniasis?


• Treat with Praziquantal or Niclosamide
• Important to distinguish between different eggs, b/c eggs of T. solia are activated by gastric juices and
Praziquantal can cause nausea as T. solia move to stomach, so must monitor patient in office during
treatment.

19. What is the significance of, and causative agents of, Cysticercosis?
• Cysticercosis is an infection of human tissues with the larvae of the pork tapeworm, Taenia solium.
• The symptoms of cysticercosis are caused by the development of cysticerci in various sites. Of greatest
concern is cerebral cysticercosis (or neurocysticercosis), which can cause diverse manifestations including
seizures, mental disturbances, focal neurologic deficits, and signs of space-occupying intracerebral lesions.
Death can occur suddenly. Extracerebral cysticercosis can cause ocular, cardiac, or spinal lesions with
associated symptoms. Asymptomatic subcutaneous nodules and calcified intramuscular nodules can be
encountered.
• Humans can be both intermediate host and main host for T. solia, not for T. saginata.

20. What is the epidemiology of Cysticercosis?


• Taenia solium is found worldwide, but particularly common in Mexico and other parts of Latin America.
• It is the most common parasitic disease of the CNS in the world, and the most frequent parasitic cause of
seizures!
• Because pigs are intermediate hosts of the parasite, completion of the life cycle occurs in regions where
humans live in close contact with pigs and eat undercooked pork.
• Taeniasis and cysticercosis are very rare in Muslim countries.
• It is important to note that human cysticercosis is acquired by ingesting T. solium eggs shed in the feces of a
human T. solium tapeworm carrier, and thus can occur in populations that neither eat pork nor share
environments with pigs.

21. What is the life cycle of Taenia solium?


• T. solium has separate adult (intestinal tapeworms) and larval (metacestode or cysticercus) forms (which is
typical of cestodes in general). Humans can host both forms of the parasite, in contrast to other cestodes.
• In the normal life cycle (described above), the larval form is found primarily in pig muscle (intermediate
host).
• Human cysticercosis follows ingestion of ova from a tapeworm carrier – typically through close personal
contact or food preparation, and carriers can autoinfect also.
• Following ingestion, the oncospheres are carried to a number of organs such as skeletal and cardiac muscle,
subQ tissue, and lungs. In most locations cause few symptoms, and just form calcific granulomas.

22. What are the clinical manifestations of Cysticercosis?


• Can be 3-15 years before patient becomes symptomatic.
• Cysticercosis involving CNS is neurocysticercosis.
• Clinical presentation, pathogenesis, management and prognosis varies widely based on location and number
of cysticerci and associated host response.
• Neurocysticercosis is typically separated into parenchymal and extra-parenchymal disease based on whether
or not there is involvement of the subarachnoid space and ventricles, but also include spinal and ocular
disease.
• Parenchymal neurocysticercosis usually presents with headaches & seizures (most common presentation)
and has a favorable prognosis.
• Neurocyticercosis due to extraparenchymal disease is more severe, often requires surgery, and can be fatal.
• Subarachnoid and ventricular diseases are often complicated by increased intracranial pressure and
hydrocephalus, so patients present with headache, dizziness, altered mental status, and visual chances due to
papilledema.
• Cysticerci can also involve the muscles
• Soft tissue calcifications are frequently found in all forms of Cysticercosis.
• Cystecerci often involve the SubQ tissues, where they present as palpable, painless and mobile cystic lesions
that can be confused with sebaceous cysts – rare, but common in SS Africa.

23. How is Cysticercosis diagnosed?


• Through a combo of epidemiology, CT scan, MRI, and serology (Western Blot is highly sensitive and
specific if 2+ cysts are present, ELISA is less specific). Serology is only 30% sensitive if only 1 cyst.

24. How is Cysticercosis treated?


• Albendazole or Praziqunatel
• Albendazole has been found to be superior in clinical studies. It works by inhibiting polymerization of
parasitic microtubules >>> decreased glucose uptake >>> eventual death.
• Corticosteroids are used to control inflammation in Neurocysticercosis, and anti-seizure meds.
• Patients with hydrocephalus often require a ventrico-peritoneal shunt to relieve pressure.
• All patients with cysticercosis should be examined for tapeworms and their family should be examined as
well.

LECTURE #4: CESTODES II - ECHINOCOCCUS

1. What is Cystic Hydatid Disease?


• Cystic Hydatid Disease is a zoonotic disease in humans and diverse mammalian species. It is caused by the
larval form of a canine tapeworm, Echinococcus granulosus.
• E. granulosus is a small tapeworm (2-7 mm in length) typically of 3 segments.
• In the natural cycle, dogs and other canines are typical definitive hosts and ungulates (sheep, goats, pigs, horses,
etc.) are intermediate hosts, which harbor the metacestode stage.
• The metacestode stage can also develop in mammals such as marsupials, rabbits, rodents, carnivores, primates and
humans. These and other hosts play a role in the transmission cycle (intermediate hosts) or are dead ends for
development (aberrant hosts).
• Important disease, b/c very common!

2. Describe the pathogenesis of an infection with E. granulosis.


• Echinococcus is caused by the metacestode stage of various strains of E. granulosus, which is a cystic structure
filled with hydatid fluid.
• About 5 days after ingestion of eggs, the metacestode is a small, bilayered vesicle.
• The cyst gradually expands and induces a granulomatous reaction in the host >>> followed by a fibrous reaction
and formation of a connective tissue layer or pericyst.
• The size of cysts in the human body vary (1-20 cm) and remain sterile.
• The initial phase of primary infection is always asymptomatic. Small, well encapsulated, non-progressive or
calcified cysts typically do not induce major pathology, and patients may remain asymptomatic for years or
permanently.
• The induction of morbidity depends on the number, size and developmental status of the cysts within an organ, the
presence of cysts on surrounding tissues and structures, the defense mechanisms of host.
• Cyst growth is generally slow.
• Hydatid fluid is highly immunogenic, and germinal membrane is very potent >>> can form new hydatid cysts if
pieces of it break off.
• Can have circulating immune complexes >>> rarely, causes Glomerulonephritis.
• Pressure necrosis
• Expanding, space-occupying lesions.

3. Where do hydatid cysts form?


• Liver (60-70%): more frequently in right lobe, usually asymptomatic. Cysts can rupture into bile duct, resulting
from obstructive jaundice and colic-like pain followed by bacterial overgrowth. Rupture into peritoneal cavity
usually leads to secondary formation of numerous peritoneal cysts. Rupture and leakage of cyst fluid also causes
erythematous rash or anaphylaxis, and rarely death.
• Lung (25%): non-complicated cysts rarely produce
symptoms. Symptomatic patients have chest pain,
Dyspnea, fever, and hemoptysis, and can be confused
with TB. Rupture of a cyst often leads to
expectoration of hydatid fluid or membranes >>>
bacterial overgrowth and lung abscess formation.
• Bone
• CNS
• If an individual has 1 cyst, look for others!

Image: Hydatid cyst in the liver. 65-75% of hydatid


cyscts are found in the liver. The cyst consists of a
parasite-derived inner germinal membrane, which secretes
an outer acellular membrane. A pericyst which is host
derived lies outside the acellular membrane.

4. What sort of symptoms are associated with infection with Echinococcus?


• Can be asymptomatic for decades
• Signs and symptoms of hydatid disease depend on organ involvement and size of cyst.
• Onset of symptoms varies from months to years.
• Eosinophilia (25%)
• Liver and biliary tract symptoms: rupture into biliary tract can lead to obstruction and pyogenic abscesses
• Fractures of bone
• CNS, cardiac and pulmonary symptoms due to rupture into lung, pericardium, etc.
• Pain is typically seen, except in liver or lung because more room to expand

5. Discuss complications of infection with Echinococcus.


• Hepatic and Biliary Tract symptoms / complications most common
• Rupture into biliary tract >>> obstruction of bile ducts >>> ascending bacterial infection >>> pyogenic abscess
• Rupture into lung, mediastinum and pericardium
• Not a benign disease.

6. What is the distribution of Echinococcus?


• E. granulosa: worldwide; prevalent in areas where dogs are used to herd sheep – South America,
Mediterranean, southern and central Russia, central Asia, Australia, Africa, and endemic to Uruguay,
Argentina, Chile, southern Brazil, and Peru.
• E. multilocularis: northern hemisphere
• E. vogeli: South and Central America
• More common in areas where uncontrolled dogs are living close to people, or unsanitary living conditions, or
in an area with uncontrolled slaughter of
infected livestock.

7. What is the morphology of Echinococcus


granulosa?
• The metacestode stage of E. granulosoa is a
cystic structure typically filled with a clear
fluid (hydatid fluid).
• It is a small vesicle (60-70 micrometers in diameter) consisting of an internal cellular layer (germinal layer),
and an outer acellular layer (laminated layer).

Echinococcus granulosus, the entire adult tapeworm consists of a scolex and 3 segments: an immature, mature and
gravid proglottid.

8. What is the life cycle of Echinococcus?


• Life cycle involves two mammalian hosts
• Adult cestode inhabits small intestine of a carvinore (definitive host) >>> proglottids containing eggs or free
eggs are released from intestinal tract of carnivore into environment.
• The eggs contain the infective oncosphere
• After oral uptake of eggs by an intermediate host >>> the larval stage or metacestode develops in this host’s
internal organs.
• Mature metacestode produces numerous protoscoleces that have the potential to develop into an adult cestode
if ingested by a suitable definitive host (such as dogs).
• The invasion of various organs (primarily lungs and liver) of intermediate hosts by metacestodes can cause
severe and even fatal disease (echinococcus).
• Humans are the accidental intermediate host.

Left image: Protoscolices are formed by the germinal membrane of the hydatid cyst and together with internal daughter
cysts, brood capsules are the hydatid sand all of which are suspended in the hydatid fluid. Right image: Echinococcus
granulosus, protoscolex note double row of hooks.
9. How do you diagnose Echinococcus infection?
• Geographic history important
• History of canine (and also possibly sheep)
• Serological tests (immunoblot)
• MRI, CT, X-ray, US

10. What are the problems with serodiagnosis?


• False positives using whole hydatid cyst fluid; could be other helminthic infection, could be cancer, could be
immune diseases such as SLE, RA.
• False negative results also occur 50% of time, particularly in lung, spleen and brain
• ELISA test good for intiital screen 60-90% sensitive, but not very specific, confirm with immunoblot 8, 16, 21
kDa bands), but this is also found in 5-10% of patients with cysticercosis.
• So, need geographic history, scans, etc. to make diagnosis.
• Monitor IgG4 for efficacy of treatment
• In seronegative patients in which you have a high suspicion of infection, do percutaneous aspiration of liver
looking for hyatid sand.

11. What is the significance of intrabiliary rupture with Echinococcus infection?


• Extremely important to diagnose intrabiliary rupture in this disease, b/c requires immediate surgery to remove
cyst.
• Endoscopic retrograde cholangiopancreatography (ERCP).
• CT
• MRI
• US

12. How do you treat Hydatid Disease?


• Surgical resection if cyst is leaking, infected or has a biliary tree infection, or >10cm in size
• Drug therapy: Albendazole primarily, rarely Mebendazole. Albendazole is more biologically active, but they are
metabolized to same compound.
• Albendazole therapy alone is unpredictable - cure rate only 30-40%
• PAIR: percutaneous aspiration injection & reaspiration.
• Protoscolicidal Treatment (spill) Albendazole and Praziquantel
• Percutaneous drainage + Albendazole is most effective treatment.

13. What is Alveolar Hydatid Disease?


• Alveolar Hydatid Disease is due to human infection with Echinococcus multilocularis.
• Humans are the abnormal, accidental, dead-end hosts of E. multicularis.
• They acquire disease by ingesting eggs shed from its final host (typically foxes).
• The primary lesion is always in liver.
• The oncosphere (embryo) hatches from egg in small intestine and is transported bias portal circulation of liver.
After hepatic localization, oncospheres undergo larval development >>> forming multilocular hydatid cysts.
• These cysts grow slowly, reproducing asexually by lateral budding, and spread through the liver and contiguous
organs to produce chronic space-occupying tumor-like lesions.
• These cysts occasionally metastasize – usually to lungs and brain.

Echinococcus multilocularis (in abdomen of shrew on right).

14. Where is E. multilocularis (aka Alveolar Hydatid Disease) found?


• Northern Europe including Alps
• Russia – very common in Siberia
• Northern US, Canada, Alaska
• Northern Japan
15. Compare the life cycles of Echinococcus granularis and E. multilocularis.

The adult Echinococcus granulosus (3 to 6 mm long) resides in the small bowel of the definitive hosts, dogs or other
canids. Gravid proglottids release eggs that are passed in the feces. After ingestion by a suitable intermediate host
(under natural conditions: sheep, goat, swine, cattle, horses, camel), the egg hatches in the small bowel and releases an
oncosphere that penetrates the intestinal wall and migrates through the circulatory system into various organs, especially
the liver and lungs. In these organs, the oncosphere develops into a cyst that enlarges gradually, producing protoscolices
and daughter cysts that fill the cyst interior. The definitive host becomes infected by ingesting the cyst-containing organs
of the infected intermediate host. After ingestion, the protoscolices evaginate, attach to the intestinal mucosa , and
develop into adult stages in 32 to 80 days. The same life cycle occurs with E. multilocularis (1.2 to 3.7 mm), with the
following differences: the definitive hosts are foxes, and to a lesser extent dogs, cats, coyotes and wolves; the intermediate
host are small rodents; and larval growth (in the liver) remains indefinitely in the proliferative stage, resulting in invasion of
the surrounding tissues. With E. vogeli (up to 5.6 mm long), the definitive hosts are bush dogs and dogs; the intermediate
hosts are rodents; and the larval stage (in the liver, lungs and other organs) develops both externally and internally,
resulting in multiple vesicles. E. oligarthrus (up to 2.9 mm long) has a life cycle that involves wild felids as definitive
hosts and rodents as intermediate hosts. Humans become infected by ingesting eggs , with resulting release of
oncospheres in the intestine and the development of cysts , , , , , in various organs.

16. Discuss the nature of infection with E. multilocularis.


• Grows slowly can be >30 years before symptoms
• Always found in liver (30% in right lobe, 60% both lobes)
• Contiguous extension
• Metastatic to lung and brain
• Clinical presentation: most common signs are:
1. hepatomegaly
2. palpable peritoneal masses
3. jaundice
• Very difficult to treat, high fatality 50-75% of patients with invasive disease

17. What is the pathogenesis of E. multilocularis?


• Contrary to the term cyst, E. multilocularis occurs as 1+ firm, solid, yellow-gray cancer-like masses that are
always primarily in liver.
• The lesions can occupy any area of the liver (including its surface) with multiple foci. The outer layer of the cyst
has an ill-defined margin that diffuses into the host tissue.
• Larval exogenous proliferation is most active at the periphery of the lesion.
• In advanced cases, the central deeper portions of the lesions can undergo extensive hepatic degeneration with
central necrosis and calcification, presenting as large, pus-filled cavities.
• 3 categories of patients:
1. Disease with unlimited proliferation
2. Proliferation is arrested in liver, lesion calcifies, and person is fine
3. Seropositive patients with no lesions (+Em2 antigen), completely asymptomatic
• Pre-existing infection confers resistance (murine)

18. What is the normal intermediate host for E. multilocularis?


Rodents

19. How is E. multilocularis diagnosed?


• Usually diagnosed in people >50 years
• MRI/CT scan showing diffuse liver disease with calcifications
• Serology: immunoblot, IHA+ in >90%
• IgG1 + IgG4 correlate with active disease

20. How is E. multilocularis treated?


• Surgical resection
• Long-term medical treatment with Albendazole is used in some patients >>> arrests growth, but won’t kill cysts.

21. Discuss some characteristics of Hymenolepsis nana.


• Hymenolepiasis is caused by Hymenolepsis nana, and is the most common cause of all cestode infections, and is
encountered worldwide, and most frequent tapeworm infection in US.
• 20 million infections worldwide
• No intermediate host is required! Internal or external autoinfection occurs. This may differ from textbooks.
Individuals can reinfect themselves, so can have large concentrations of eggs and 1000’s of tapeworms >>>
clinical symptoms.
• In temperate areas its incidence is higher in children and institutionalized groups.
• Hymenolepis diminuta, while less frequent, has been reported from various areas of the world.
• Hymenolepis nana is the smallest adult tapeworm to infect humans – 25-30mm in length x 0.80-1.0 mm in width.
• The entire chain of proglottids from the anterior immature to the distal gravid segments often consists of 175-220
segments.
• The minute rounded scolex possesses 4 sucking disks and a short armed retractable rostellum.
• The infective eggs are liberated when the most distal proglottids disintegrate and are found in stool. They are
released by gradual disintegration of the terminal gravid proglottids and are infective immediately when passed in
the feces.
• If eggs are ingested, the oncosphere is liberated in host’s small intestine and penetrates villi >>> to become
cysticercoid larvae.
• In 96 hours the larvae re-enters the lumen to attaché to SI villus by its scolex, usually at a more distal location.
• In 10-20 days it matures and eggs can be found in feces.
• There is evidence of hyperinfection or autoinfection when ova liberated in the SI spontaneously hatch and
immediately penetrate intestinal villi to undergo a new development cycle.
• Human is definitive host. Unlike most other tapeworms that infect humans, no intermediate host is required.
Left image: Hymenolepsis nana scolex with 4 suckers & armed retractable rostellum. Right image: H. nana proglottids.

22. What is the life cycle of H. nana?

Eggs of Hymenolepis diminuta are passed out in the feces of the infected definitive host (rodents, man) . The mature
eggs are ingested by an intermediate host (various arthropod adults or larvae) , and oncospheres are released from the
eggs and penetrate the intestinal wall of the host , which develop into cysticercoid larvae. Species from the genus
Tribolium are common intermediate hosts for H. diminuta. The cysticercoid larvae persist through the arthropod's
morphogenesis to adulthood. H. diminuta infection is acquired by the mammalian host after ingestion of an intermediate
host carrying the cysticercoid larvae . Humans can be accidentally infected through the ingestion of insects in precooked
cereals, or other food items, and directly from the environment (e.g., oral exploration of the environment by children).
After ingestion, the tissue of the infected arthropod is digested releasing the cysticercoid larvae in the stomach and small
intestine. Eversion of the scoleces occurs shortly after the cysticercoid larvae are released. Using the four suckers on the
scolex, the parasite attaches to the small intestine wall. Maturation of the parasites occurs within 20 days and the adult
worms can reach an average of 30 cm in length . Eggs are released in the small intestine from gravid proglottids that
disintegrate after breaking off from the adult worms. The eggs are expelled to the environment in the mammalian host's
feces. (www.cdc.gov)
23. What is the clinical presentation of infection with H. nana?
• If patient only has a few tapeworms probably will be asymptomatic, but if they have a lot of worms, will have the
following symptoms:
• Headaches
• Dizziness
• Anorexia
• Pruritis ani
• Periodic diarrhea
• Abdominal pain (85%)
• Restlessness and irritability (65%)
• Eosinophilia (<5%)
• Seizures (<1%)

24. What is the treatment of choice for H. nana?


• Praziquantel

25. How are public lice acquired?


• Through close personal contact.
• Not from toilet seats, or any other casual contact.
• Head lice can be transmitted from child to child through wrestling, etc.

26. How are lice treated?


• 1% Permethrin (Nix), topical treatment
• 0.5% Malathion (Ovide) topical
• Pyrethrins with pipernyl butoxide (Rid) topical treatment
• Ivermectin (Stromectol) – insecticide and powerful antihelminthic, taken orally, used for lice if first treatments
don’t work. May soon be first line of defense.

LECTURE #5: NEMATODES I – ASCARIS, HOOKWORM, VLM, CLM

1. What organisms cause hookworm disease in humans?


• Necator americanus
• Ancylostoma duodenale

2. What is the hallmark of hookworm infection?


• Iron deficiency anemia due to chronic blood loss
• A single hookworm can suck 0.3 ml of blood per day, and
often person is infected with multiple worms and is
concurrently malnourished.
• Uses very muscular pharyngeal complex to pump blood
Image: Hookworm sucking blood

3. What is the distribution of A. duodenale and N.


americanus?
• Big global health problem, infect 1 billion people
• Impacts health of mothers and children >>> malnutrition
A. A. duodenale
• Tropical and subtropical Asia and Africa
• Middle East
• North Africa
• Southern Europe
B. N. americanus
• Tropical and subtropical Asia and Africa
• New world – Appalachia, Alabama

4. Describe the morphology of Hookworms.


• Adult hookworms are small, cylindrical/spindle-shaped creamy white nematodes
• Bilaterally symmetrical, unsegmented, acellular outer body wall w/ underlying hypodermis containing longitudinal
muscle for locomotion.
• Cuticles of different kinds smooth, spiny or striated
• No respiratory or circulatory systems, but do have primitive digestive system (mouth, buccal cavity, esophagus,
rectum, anus) and nervous system
• Body cavity not lined by epithelial cells >>> pseudocoelom
• Males 5-11 mm long, females bigger 9-13 mm long
• Identification is based on length, number and arrangement of teeth or cutting plates, length of esophagus, and
morphology of genital primordial.
• N. americanus has a head curved opposite to the curvature of the body, giving a hooked appearance to the anterior
end. The buccal capsule has a pair of dorsal and ventral semilunar cutting plates.
• A. duodenale has a head which continues in the same direction as the curvature of the body. The buccal capsule
has 2 pairs of curved ventral teeth or nearly the same size.

Necator americanus: showing male copulatory bursa

5. What is the life cycle of the hookworm?


• Most actually live freely in soil, rather than as parasites.
• Females lay millions of eggs, but very few mature
• Complicated life cycles with obligatory intermediate hosts
• Life stage infective for people is filariform larvae negatively geotrophic and positively pheotropic
• Like to enter and live in the feet, groundage
• Vertical transmission through milk and orally may be possible
Eggs are passed in the stool , and under favorable conditions (moisture, warmth, shade), larvae hatch in 1-2 days in soil.
The released rhabditiform larvae (250-300 micrometers in length with a bluntly rounded anterior end and a long, narrow
buccal cavity) grow in the feces and/or the soil , and after 5 to 10 days (and two molts) they become filariform larvae
(third stage) that are infective . These infective larvae can survive 3-4 weeks in favorable environmental conditions. On
contact with the human host, the larvae penetrate the skin and are carried through the veins to the heart and then to the
lungs. They penetrate into the pulmonary alveoli, ascend the bronchial tree to the pharynx, and are swallowed . The
larvae go through stomach to the small intestine, where they reside and mature into adults. Adult worms live in the lumen
of the upper half of the small intestine, where they attach to the intestinal wall with resultant blood loss by the host .
Most adult worms are eliminated in 1 to 2 years, but longevity records can reach several years. Some A. duodenale larvae,
following penetration of the host skin, can become dormant (in the intestine or muscle). In addition, infection by A.
duodenale may also occur by the oral and transmammary route. N. americanus, however, requires a transpulmonary
migration phase. (www.cdc.gov)

Hookworm egg
6. What are the clinical manifestations of infection with N. americanus or A. duodenale?
• Infections last 5-10 years, but no auto-infectious cycle (infection continuous even outside of endemic area)

7. How are hookworm infections treated?


• Mebendazole
• Albendazole
• Most important thing is education >>> teaching people to wear shoes, and not to use human feces in gardening
• There may be a vaccine available soon based on Ancylostoma secreted proteins (ASP-1, ASP-2)
• Ancylostoma caninum infection >>> eosinophilic gastroenteritis

9. What are the characteristics of Ascaris lumbricoides, and how is it diagnosed?


• 1 billion infections worldwide
• Distribution: worldwide in tropics w/ moist and shady soils
• Pollen in the stool is indicative of Ascaris infection
• Egg is both diagnostic and infective stage!
• Live for about 2 years

Left image: Ascaris lumbricoides adult worms. The male is


smaller (120-250mm x 3-4mm) than the female (200-400mm x 5-6mm), and the male's posterior end is frequently coiled or
recurved. Right image: Ascaris lumbricoides ovum: The 40-60µm eggs have a characteristic mamilated outer coat and
thick hyaline shell.

10. What is the morphology of Ascaris?


• Cylindrical pink or cream-colored adult worm tapers at both ends.
• Male is smaller (120-250mm x 3-4mm) than female (200-400mm x 5-6mm)
• Male’s posterior end is slightly coiled
• Female produces 200,000 eggs per day

Ascaris lumbricoides decorticate eggs, note: developing larvae

11. What is the life cycle of Ascaris lumbricoides?


• Needs warm, moist shaded soil to mature
• Infectious eggs are ingested and larvae hatch in jejunum >>> penetrate intestinal wall >>> migrate via hepatic
venules to right heart and pulmonary circulation >>> break into alveolar spaces and undergo 2 additional molts.
Most larvae will have reached the lungs by 2 weeks after ingestion of eggs >>> from alveoli, larvae ascend to
trachea and are swallowed >>> undergo another molt in intestine >>> develop into adults. Incubation period after
being swallowed by human is 2-3 months.
• Very hardy eggs, can survive months without oxygen, etc.

Adult worms live in the lumen of the small intestine. A female may produce approximately 200,000 eggs per day, which
are passed with the feces . Unfertilized eggs may be ingested but are not infective. Fertile eggs embryonate and become
infective after 18 days to several weeks , depending on the environmental conditions (optimum: moist, warm, shaded
soil). After infective eggs are swallowed , the larvae hatch , invade the intestinal mucosa, and are carried via the
portal, then systemic circulation to the lungs . The larvae mature further in the lungs (10 to 14 days), penetrate the
alveolar walls, ascend the bronchial tree to the throat, and are swallowed . Upon reaching the small intestine, they
develop into adult worms . Between 2 and 3 months are required from ingestion of the infective eggs to oviposition by
the adult female. Adult worms can live 1 to 2 years. (www.cdc.gov)

12. What is the clinical presentation of infection with Ascarcis?


• Pulmonary hypersensitivity that manifests as asthma
• Acute symptoms: usually in children
1. urticaria
2. bronchospasm
3. pneumonia
• It can cause intestinal obstruction and abdominal pain, particularly in a child
• Typically asymptomatic in adults
• It can cause peritonitis, pancreatitis, inflammation in biliary tree
• Loeffler’s Syndrome: prolonged eosinophilia and cardiomyopathy, often caused by crack cocaine use.
13. What are the complications of infection with Ascarcis?
• Abdominal pain
• Nausea, vomiting
• Intestinal obstruction (children) >>> rarely, leads to volvulus
• Peritonitis
• Biliary and pancreatic obstruction

14. How is an Ascarcis infection treated?


• Albendazole
• Mebendazole
• Ivermectin
• Reduce infections by education on sanitary waste disposal

15. What is the significance of Cutaneous Larva Migrans?


• Cutaneous Larva Migrans classic for creeping eruption
• Dermatitis caused by invasion of skin by larval nematodes: dog
hookworm, typically Ancylostoma Braziliensis, found in sandy
areas in the Caribbean, southeast US, and elsewhere. Also caused
by A. caninum.
• Dermatitis caused by invasion of skin by larvae of dog and cat
hookworm, but doesn’t penetrate any deeper than epidermis
• Life cycle: A. braziliensis & A. caninum inhabit the intestines of
dogs and cats >>> embryonated eggs are passed in stool >>> if
right temp & humidity >>> hatch and release rhabditiform larvae
in 1-2 days >>> after feeding, growing & molting >>> become
infective filariform larvae (1 week) >>> larvae penetrate skin of
humans no deeper than epidermis.
• Distribution: warm tropical or substropical regions, SE US
• Humans are aberrant host.
• More common in children, bare skin contact with soil is necessary
for transmission
• Clinical manifestations: itching red papule w/ serpiginous track
from worm >>> inflamed edematous tissue; not deadly, but very
itchy >>> scratch becomes infected with bacteria
• Diagnosis: based on history and serpiginous track
• Treatment:
1. Albendazole
2. Ivermectin
3. Topical Thiabendazole

15. What is Toxicariasis?


• Dogs and cats have their own ascaris known as toxocara canis and
toxicara cati; Tocicariasis is infection with embryonated eggs of dog
ascarid T. canis or less often cat ascaris, T. cati.
• People get infected with a worm, in which humans are not the
definitive host, so they don’t develop into adult in human, the worm
remains in larval form and cause a spectrum of disease ranging from
asymptomatic to VLM and OLM.
• VLM (Visceral Larval Migrans)
• OLM (Oculolarval migrans): eye can be involved
• Soil can be contaminated with toxocara canis eggs
• Cats, dogs, and PICA
• Eosinophilic granulomas

Image: Toxocara eggs (not seen in humans only in cats or dogs).


16. What is the life cycle of Toxocaris canis or cati?
• Adult T. canis & T. cati live in dog or cat intestines >>> eggs shed in stool >>> 2-3 weeks in warm, moist soil and
appropriate temps
• Tranmission to humans occurs via ingestion of embryonated eggs directly from soil or from contaminated hands
>>> eggs hatch and release larvae in proximal small intestine, where they penetrate the mucosa and are carried to
the liver via portal circulation >>> some larve remain in liver forming granulomas >>> other go to lungs >>>
larvae enter the circulation and are carried to various parts of the body until they reach a vessel too small for
passage >>> penetrate into surrounding tissue.

Toxocara canis accomplishes its life cycle in dogs, with humans acquiring the infection as accidental hosts. Infective eggs
are excreted by dogs. Humans are paratenic hosts who become infected by ingesting infective eggs in contaminated soil.
After ingestion, the eggs yield larvae that penetrate the intestinal wall and are carried by the circulation to a wide variety of
tissues (liver, heart, lungs, brain, muscle, eyes). While the larvae do not undergo any further development in these sites,
they can cause severe local reactions that are the basis of toxocariasis.
17. Compare and contrast the clinical presentations of VLM with those of OLM.
A. VLM
• VLM is usually found in toddlers (ages 2-4) who eat soil
• Marked inflammatory immune response to numerous migrating larvae in liver and other tissues.
• Get febrile systemic illnesses manifested by fever and chills, fleeting infiltrates in chest, may have seizures,
myocarditis, or hepatitis
• Very high levels of eosinophils (30-50%), leukocytosis, fever, hepatomegaly, hypergammaglobulinemia,
Isohemagglutinins are elevated, ery high IgE
• + ELISA test, serology is only way to make definitive diagnosis, because human is not host, won’t see eggs in
stool.
• High dose may end up as VLM, low dose of eggs ingested more likely to have OLM.
B. OLM
• School-age children and adults
• Sitting in school has to strain eyes to see
• Ocular lesions, strabismus, retinal scar on opthalmoscopic exam
• Diagnosis: + ELISA test
• Won’t have eosinophilia, isohemagglutinins or systemic
manifestations

Image: Toxocariasis in the eye causing chororetinitis or ocular larva


migrans (OLM).

18. How do you treat VLM and OLM?


• Eliminate source of infection VLM
• Symptomatic treatment of VLM and OLM
• Surgery or laser therapy for OLM
• Antihelminthics:
1. Diethylcarbazamine
2. Thiabendazole
3. Albendazole

19. What is Balyis ascaris?


• is an ascaris of raccoons.
• Raccoons either enter the house or kids play in yard and eat raccoon droppings, thereby ingesting the eggs >>>
eggs hatch >>> infect kids >>> neural larval migrans
• Typically infects CNS
• there is no good treatment, so kids often die from this

20. What is Angiostrongylus?


• It is a cause of eosinophilic meningitis.
• People get infected by eating snails, crabs or prawns >>> larva comes out and enters the brain >>> eosinophilic
meningitis.
• Humans are accidental hosts.
• No serology
• No therapy
• Diagnosis made by finding larvae in CSF, history of eating raw food, travel to endemic areas.

LECTURE #6: NEMATODES II

Intestinal Nematodes
• Oral-fecal: Enterobius vermicularis (pinworm), Trichuris trichuria (whipworm), no migration in the lung
• Oral-fecal with migration to lungs: Ascaris lumbricoides
• Soil-migration to lungs: N. americanis and A. duodenales (hookworm), and Strongyloides sterocoralis, patients
can get them from walking barefoot
• All are treated with Albendazole
Infection with Enterobius vermicularis (Pinworm)
• Common worldwide: including temperate and colder climates, particularly noted in cooler climates b/c worms are
more unusual in N. America, found in all socio-economic groups.
• Transmission: hand to mouth transmission; egg is excreted in fairly mature form and can be passed from person-
to-person, rather than requiring incubation in warm, moist soil. Humans are only host.
• Distribution: most common helminthic infection in US and W. Europe. Greatest prevalence amongst school-aged
children, high population density, indoor conditions.
• Pinworm season is typically in middle of winter, b/c kids are cuddling together or sleeping together more often

Pinworm Morphology
• Female 8-13 mm in length (can be seen with naked
eye) much larger than male, female has pin tail, male
does not
• So as physician look for eggs on perineal skin, rather
than checking stool sample
• Pinworms have ala or ventral wings
• Round = nematode
• Eggs are flattened on one side and convex on other
side, have thick albuminous layer that can get
attached to dust, they mature and are infective within
6 hours, and remain infective for several days,
infectious egg contains first stage larvae. Look like
coffee bean.

Top image: Enterobius vermicularis (pinworm) adult male demonstrating cephalic alae. Lower images: Enterobius
vermicularis (pinworm) eggs on scotch tape test. The eggs are elongate-ovoidal, compressed laterally, flattened on one
side and measure 50-60 x 20-30µm.

Enterobius Vermicularis Life Cycle


• Eggs contain larvae that must go through further maturation before becoming infective, atmospheric oxygen acts
as a stimulus to development.
• At body temp, eggs become infective within 6 hours, and lose infectivity 1-2 days under dry warm conditions.
Survival better with low temp, humid conditions.
• Infestation follows ingestion of eggs, which usually reach mouth on soiled hands or contaminated food. After
being ingested, eggs hatch in upper SI >>> liberating larvae which migrate to Ileum >>> molt twice along way to
become adults >>> copulation occurs in lower SI >>> adult females settle in cecum, appendix, or adjacent areas of
ascending colon.
• Adult females live up to 13 weeks and most patients have a few to 100s of worms.
• Gravid female 17,000 eggs >>> detaches from cecal mucosa >>> migrates down large bowel >>> passes out of
anus to perineal skin >>> migration with trail of eggs on skin
• Egg goes into environment and takes 6-8 hours to embryonate, so eggs can be shed into bed sheets and then
another child sleeps in that same bed can get infection by the next night

Life Cycle:
Eggs are deposited on perianal folds . Self-infection occurs by transferring infective eggs to the mouth with hands that
have scratched the perianal area . Person-to-person transmission can also occur through handling of contaminated
clothes or bed linens. Enterobiasis may also be acquired through surfaces in the environment that are contaminated with
pinworm eggs (e.g., curtains, carpeting). Some small number of eggs may become airborne and inhaled. These would be
swallowed and follow the same development as ingested eggs. Following ingestion of infective eggs, the larvae hatch in
the small intestine and the adults establish themselves in the colon . The time interval from ingestion of infective eggs
to oviposition by the adult females is about one month. The life span of the adults is about two months. Gravid females
migrate nocturnally outside the anus and oviposit while crawling on the skin of the perianal area . The larvae contained
inside the eggs develop (the eggs become infective) in 4 to 6 hours under optimal conditions . Retroinfection, or the
migration of newly hatched larvae from the anal skin back into the rectum, may occur but the frequency with which this
happens is unknown.

Clinical Presentation of Pinworm Infection


• Doesn’t cause a lot of pathology b/c no attachment; instead, they glide on top of the colonic mucosa >>> doesn’t
cause eosinophilia (which helps fight parasites).
• Cause perineal eczematous dermatitis and pruritis particularly at night, b/c female migrates out of anus at night.
• Scratching contaminates fingers and can lead to autoinfection.
• Can see folliculitis in adults.
• Very rare complication: female migrates into vagina >>> causes vaginal pruritis & UTIs particularly in
pubescent females.
• Often major psychological impact on parents
• No association with thumbsucking
• Close association with Dientamoeba fragilis which is a protozoa, possibly pinworm carries the protozoa with it.
• Appendicitis is common and pinworm is common in children, but no connection between the two.

Diagnosis of Enterobius Vermicularis


• Scotch tape affixed to pernieal region as soon as kids awaken (before they go to the bathroom).
• Look at tape under microscope and see eggs.

Treatment of Enterobius Vermicularis


• Mebendazole: same mechanism as Albendazole.
• Albendazole: is much better absorbed than Mebendazole, so you get higher levels with fewer doses. Mechanism
of action is bind to tubulin – do not use in pregnant women!
• Pyrantel Pamoate – they don’t use it here in clinic, M-oxyphenol derivative that functions as a depolarizing
neuromuscular blocking agent >>> spastic paralysis of worms.
• Therapy should be repeated within 2 weeks.
• Other household members should be examined and treated.

Infection With Trichuris Trichuria (Whipworm)


• One of the most common helminthic infections (vying with ascaris) in the world – 900 million infections. Can be
milder forms.
• Distribution: found worldwide, and in US in rural communities of southern US, similar distribution as Ascaris
• Eggs ingested hand-to-mouth
• Transmission: from raw fruits and vegetables, in places where people fertilize produce with human feces
• Eggs are passed in feces, they need to mature in soil for 10-14 days
• About 3 months required for patent infection in human after ingestion of eggs

Morphology of Trichuris Trichuria


• 3-5 cm adults
• Anterior 3/5 are threadlike, posterior 2/5 more substantial with reproductive organs.

Life Cycle of Trichuris Trichuria


• ingestion of embryonated egg
• larvae emerge in intestine >>> penetrate the crypts of Lieberkuhn and migrate within the mucosal epithelium.
• On molting to adult form >>> posterior end is projected out of epithelium leaving a threadlike anterior portion
embedded in syncytium created from epithelial cells.
• Mature female produces 2000-6000 eggs per day >>> eggs are passed in feces >>> mature in soil for 10-14 days
• 3 months required for infection be produced from ingested eggs.
• Adult worms may persist for years.

Life Cycle:
The unembryonated eggs are passed with the stool . In the soil, the eggs develop into a 2-cell stage , an advanced
cleavage stage , and then they embryonate ; eggs become infective in 15 to 30 days. After ingestion (soil-
contaminated hands or food), the eggs hatch in the small intestine, and release larvae that mature and establish
themselves as adults in the colon . The adult worms (approximately 4 cm in length) live in the cecum and ascending
colon. The adult worms are fixed in that location, with the anterior portions threaded into the mucosa. The females begin
to oviposit 60 to 70 days after infection. Female worms in the cecum shed between 3,000 and 20,000 eggs per day. The
life span of the adults is about 1 year.

Clinical Presentation of Trichuriasis


• Local eosinophilic infiltrate in mucosa in milder form
• In very heavy infections, people can get chronic diarrhea,
dysentery syndrome, abdominal pain, pro-lapsed rectum,
bowel wall edema and mucosa may bleed >>> anemia blood
loss due to friable mucosa
• Highest worm burden found in young children: children can
get chronic colitis with growth retardation and it may impair
cognitive abilities (can interfere with kids’ schooling).
Children with PICA or short stature may have had whipworm.
• Worms don’t ingest blood, but can get anemia from bleeding
due to mucosal damage.
• Can see clubbing.

Diagnosis of Trichuriasis
• Take 3 stool samples from 3 different days and look for eggs
under microscope.

Treatment of Trichuriasis
• Put the rectum back in place, does not require surgery
• Albendazole
• Mebendazole

Infection with Strongyloides Stercoralis


• Unique among helminthes due to Autoinfection, can maintain persistent infection for years.
• Can get active infection 60+ years after exposure >>> so when doing a history you should always ask about where
person was born, any travel, and animal (particularly rodents in the house) exposure.
• World distribution: occurs in SSA, Asia, Carribean, Mexico, southern and southeastern US, South America,
Puerto Rico (heavily endemic)
• Transmission: walking barefoot penetrates skin
• Immunosuppressed (steroids or infection with HTLV-1) can get hyperinfection syndrome with high morbidity.
• Symptoms: cough, wheezing, bronchospasm (due to migration of worm through lungs).
• Loffler’s Syndrome
• Can get Duodenitis >>> peptic ulcer-like symptoms

Life Cycle of Strongyloides Stercoralis


• Get it from the fecally-contaminated soil when walking barefoot, penetrates unbroken skin, female filariform
larvae (3rd stage) migrate through venous system to lungs >>> penetrate alveoli >>> trachea >>> glottis >>>
swallowed >>> burrow into Small Intestine wall and releases eggs there >>> mature eggs are released into lumen
of bowel >>> egg releases rhabditiform larvae which goes out into stool >>> molts >>> becomes filariform larvae
(which has the ability to penetrate unbroken skin and bowel wall, and can then go through liver, to lungs, and molt
and continue).
• Autoinfection possible, so you can live with this infection for years and years after leaving endemic area.
• Infective stage: filariform larvae.
The Strongyloides life cycle is complex among helminths with its alternation between free-living and parasitic cycles, and
its potential for autoinfection and multiplication within the host. Two types of cycles exist:
Free-living cycle: The rhabditiform larvae passed in the stool (see "Parasitic cycle" below) can either molt twice and
become infective filariform larvae (direct development) or molt four times and become free living adult males and
females that mate and produce eggs from which rhabditiform larvae hatch . The latter in turn can either develop
into a new generation of free-living adults (as represented in ), or into infective filariform larvae . The filariform
larvae penetrate the human host skin to initiate the parasitic cycle (see below) .
Parasitic cycle: Filariform larvae in contaminated soil penetrate the human skin , and are transported to the lungs where
they penetrate the alveolar spaces; they are carried through the bronchial tree to the pharynx, are swallowed and then reach
the small intestine . In the small intestine they molt twice and become adult female worms . The females live
threaded in the epithelium of the small intestine and by parthenogenesis produce eggs , which yield rhabditiform larvae.
The rhabditiform larvae can either be passed in the stool (see "Free-living cycle" above), or can cause autoinfection .
In autoinfection, the rhabditiform larvae become infective filariform larvae, which can penetrate either the intestinal
mucosa (internal autoinfection) or the skin of the perianal area (external autoinfection); in either case, the filariform larvae
may follow the previously described route, being carried successively to the lungs, the bronchial tree, the pharynx, and the
small intestine where they mature into adults; or they may disseminate widely in the body. To date, occurrence of
autoinfection in humans with helminthic infections is recognized only in Strongyloides stercoralis and Capillaria
philippinensis infections. In the case of Strongyloides, autoinfection may explain the possibility of persistent infections for
many years in persons who have not been in an endemic area and of hyperinfections in immunodepressed individuals.

Clinical Presentation of Strongyloides Infection


• Skin penetration by filariform larvae >>> pruritic maculopapular eruption, or rapidly migrating linear urticaria
called larvae currens. Creeping eruptions.
• Veterans 30-40 years later
• Abdominal symptoms: epigastric pain, watery diarrhea, abdominal pain when worms penetrate intestines
• Pulmonary symptoms: wheezing, cough, SOB if worms migrate through lungs
• Loffler’s Syndrome
What is hyperinfection syndrome?
• Eosinophils are crucial for fighting this infection. If suppress eosinophils through steroid treatment (for
lymphphoma) rhabdidiform larvae become filariform larvae >>> hyper-infection syndrome
• Loss of cellular immunity allows for conversion of rhabtidiform larvae to filariform larvae with widespread
dissemination through entire body
• Fever
• Vomiting (due to edematous bowel)
• Diarrhea (due to edematous bowel)
• Pulmonary infiltrates: because lots of larvae maturing in lung
• Hypoxia
• Gram negative sepsis
• Gram negative meningitis: go into meninges and bring coliforms with them, if you see adult with Gram negative
meningitis think of Strongyloides.
• Babies and patients with catheters in brain get gram negative infections in CSF
• HTLV-1 infection: retrovirus very common in Carribean, may be due to a cytokine issue and they are unable to
make eosinophils.
• High mortality rate if untreated.

Diagnosis of Strongyloidiasis
• Rhabtidiform larvae in stool, obtain several stool samples.
• String test (outdated) / duodenal aspirate.
• Serology – IgG4 to strongyloides, if see patient with slight eosinophilia get serology.
• Check sputum and stool for possible Hyper-infection.

Treatment of Strongyloidisis
• Thiabendazole – 95% cure rate
• Ivermectin – cidal

Which worms cause Loffler’s Syndrome?


• Ascaris
• Strongyloides
• Pinworm – does not
• Whipworm – does not

LECTURE #7: NEMATODES III

1. What organisms cause Filariasis?


• Wucheria bancrofti
• Brugia malayi
• Loa loa
• Onchocerca volvulus
• Mansonella species

1. What is Dracunculosis?
• Dracunculosis (also called Guinea Worm disease) is a painful, incapacitating disease caused by Dracunculosis
medinensis.
• Transmission: by drinking water containing cyclopoid copepods, freshwater microcrustaceans that harbor
infective larvae.
• This nematode lives in connective and subcutaneous tissue of humans.
• The female worm emerges through the skin to discharge its larvae into water.
• The clinical course is characterized by allergic prodromal symptoms and cutaneous ulceration.
• Most infections are limited to a single worm.

2. What is the morphology of Dracunculus medinensis?


• The adult female worm is creamy white, slender cord, and is one of the longest nematodes (up to 1 meter), and 2
cm wide.
• Adult males rarely seen because they don’t live long after mating. Male is half size of female.

3. What is the life cycle of Dracunculus medinensis?


• After an incubation period of 10-14 months, female worm migrates
to subQ tissue (usually in feet or legs) >>> painful blister >>> when
immersed in water, blister bursts and female worm protrudes through
skin >>> expelling 1000’s of larvae into the water by contracting her
uterus.
• Larvae are ingested by intermediate host, copepod (a Cyclops
microcrustacean)
• Humans become definitive host when they drink water contaminated
by Cyclops infected with 3rd stage larvae >>> liberated larvae
penetrate human intestinal tract wall >>> migrate into abdominal or thoracic cavity >>> develop into adults >>>
mate after 3 months >>> female continues to mature until its body consists almost entirely of a coiled distended
uterus containing millions of rhabditiform larvae >>> female begins extensive migration that ends at site of
emergence.

Humans become infected by drinking unfiltered water containing copepods (small crustaceans) which are infected with
larvae of D. medinensis . Following ingestion, the copepods die and release the larvae, which penetrate the host stomach
and intestinal wall and enter the abdominal cavity and retroperitoneal space . After maturation into adults and
copulation, the male worms die and the females (length: 70 to 120 cm) migrate in the subcutaneous tissues towards the skin
surface . Approximately one year after infection, the female worm induces a blister on the skin, generally on the distal
lower extremity, which ruptures. When this lesion comes into contact with water, a contact that the patient seeks to relieve
the local discomfort, the female worm emerges and releases larvae . The larvae are ingested by a copepod and after
two weeks (and two molts) have developed into infective larvae . Ingestion of the copepods closes the cycle .
4. What is the distribution of Dracunculus medinensis?
• Africa, Asia, Yemen and Saudi Arabia

5. What are the clinical manifestations of Dracunculosis?


• In most patients, first sign is formation of a large, reddish paule 2-7 cm in diameter with a vesicular center and an
indurated margin. This is where the cephalic end of worm is approaching skin.
• Formation of blister is accompanied by intense pruritis and burning pain >>> patient immerses affected body part
in water for relief.
• After 3-5 days, the blister ruptures and discloses small superficial ulcer - at its center is a hole with worm’s head.
• In response to contact with fresh water, a loop of uterus prolapses through the hole and releases larvae into water.
• The principal complication is secondary bacterial infection – this is serious and can cause tetanus or cellulitis.
• If in joins >>> guinea worm arthritis.

6. How is a diagnosis of Dracunculosis made?


• Once blister has broken, diagnosis can be made by placing cold water on the ulcer or ethyl chloride near it >>>
this results in release of larvae which can be viewed under microscope.

7. How is Dracunculosis treated?


• Winding each emerging worm onto a small stick of a few centimeters each day is useful as long as it is begun
when the worm first emerges.
• Surgical removal of worm.
• Treat with Benzimidazoles:
Metronidazole
Niradazole
Thiabendazole
Mebendazole

8. What is Loiasis?
• Loiasis is caused by infection with the filarial (“thread-like”)” nematode Loa loa.
• The adult worms migrate through the subQ tissue causing intermittent “calaber swellings” and sometimes
beneath the conjunctiva (eye worm).
• Microfilariae are found in the peripheral blood during the day.
• Distribution: endemic to rain forests of Central and West Africa; in some parts 100% infection rates.
• Transmission: flies of genus Chrysops or Deer Fly, which bites in shaded areas during the day.
• L3 larvae are infective form.

9. What is the morphology of Loa loa?


• The larger, female organisms are 60 mm by 500 micrometers; males are 35mm by 300 micrometers in size.
• The circulating microfilaria are 300 x 7 micrometers; the infective larvae in the fly are 200 x 30 micrometers.

10. What is the life cycle of Loa loa?


• The life cycle of Loa loa is identical to that of Onchocerca except that the vector for this worm is the deer fly.
• Day biting flies of the genus Chrysops pick up the microfilariae of Loa loa in their blood meals.
• The ingested microfilariae molt twice in the fly >>> develop into infective L3 filariform larvae >>> when flies
bite new host, larvae infect new host and develop into adult worms.
• Adult worms are thin and transparent and migrate through subQ tissue.
• Microfilariae are released into host bloodstream where they are most plentiful between the hours of 10am-2pm.
• It is the presence of a sheath and 3+ terminal nuclei that distinguish the microfilariae of Loa loa from W.
bancrofti and Mansonella perstans.
The vector for Loa loa filariasis are flies from two species of the genus Chrysops, C. silacea and C. dimidiata. During a
blood meal, an infected fly (genus Chrysops, day-biting flies) introduces third-stage filarial larvae onto the skin of the
human host, where they penetrate into the bite wound . The larvae develop into adults that commonly reside in
subcutaneous tissue . The female worms measure 40 to 70 mm in length and 0.5 mm in diameter, while the males
measure 30 to 34 mm in length and 0.35 to 0.43 mm in diameter. Adults produce microfilariae measuring 250 to 300 μm
by 6 to 8 μm, which are sheathed and have diurnal periodicity. Microfilariae have been recovered from spinal fluids, urine,
and sputum. During the day they are found in peripheral blood, but during the noncirculation phase, they are found in the
lungs . The fly ingests microfilariae during a blood meal . After ingestion, the microfilariae lose their sheaths and
migrate from the fly's midgut through the hemocoel to the thoracic muscles of the arthropod . There the microfilariae
develop into first-stage larvae and subsequently into third-stage infective larvae . The third-stage infective larvae
migrate to the fly's proboscis and can infect another human when the fly takes a blood meal . (www.cdc.gov)

11. What are the clinical manifestations of Loiasis?


• With the exception of eye involvement, clinical signs and symptoms are more common in visitors to Loa endemic
areas, b/c they have a heightened immune response (natives do not).
• Recurrent episodes of localized angioedema or caliber swellings (may not be seen in people in endemic areas due
to lack of immune response).
• Subconjuctival migration of eyeworm >>> transient swelling of lid and intense conjunctivitis.
• Mostly resolves spontaneously, but rare cases of retinal artery occlusion and macular retinopathy, due to aberrant
migration of adult parasite
• Serious complications: meningoencephalitis in patients with high circulating microfilariae. CNS involvement
ranges from mild headache and meningisimus to coma and death.
• Renal involvement common: hematuria and proteinuria are commonly seen in loiasis and may be due to
immune complex glomerulonephritis or mechanical trauma resulting from filtration of large numbers of
microcilariae.
• High eosinophilia
12. How is Loiasis diagnosed?
• Demonstration of sheathed microfilariae (still have portion of egg around them) with nuclei extending to the tip
of the tail or idenitification of adult worm removed from subQ tissue is diagnostic.
• Serology is available (unlike other filarial diseases).

13. How is Loiasis treated?


• Diethylcarbamazine (DEC) – will kill larvae and adults and is usually curative, but may require multiple courses.
• A serious complication of treatment with DEC is meningoencephalitis, particularly in patients with high
circulating microfilariae. Dying worms migrate and cause serious allergic reactions. In order to avoid this, treat
preemptively with steroids.
• Ivermectin and Albendazole may be effective

14. What is Onchocerciasis?


• Onchocerciasis is known as River Blindness, and is an infection of
humans caused by the filarial nematode Onchocerca volvulus.
• The parasite is transmitted by black flies of genus Simulium.
• Infection may be asymptomatic, but heavy infections are associated
with disease affecting skin and eyes, and in severe cases can lead to
disfigurement of skin and visual impairment or even blindness.
• Distribution: 37 countries in central Africa and northern South
America, and Arabian peninsula; particularly Congo, Sierra Leone,
Venezeula & Guatemala.
• World’s second leading cause of blindness.
• Larvae are infective form.

15. What is the morphology of Onchocerca volvulus?


• Adult female onchocerca measure 50 cm by 300 micrometers, they are long and thin, and become encapsulated in
subQ nodules (onchocercomas).
• Male worms are much smaller.
• Infective larvae of O. volvulus are 500 micrometers by 25 micrometers (figure 21).

Onchocerca volvulus microfilaria in subcutaneous tissue. The microfilaria are unsheathed in the tissues and depending on
how they are prepared for measurement are 315-360µm long by 6µm. The anterior and posterior ends are nuclei-free.

16. What is the life cycle of Onchocerca volvulus?


• The life cycle of O. volvulus is identical to that of Loa loa except that the
vector for this worm is the black fly, rather than the deer fly.
• Infection transmitted by black flies (genus Simulim), which inoculate
infective larvae into host.
• The infective larvae undergo 2+ molts over the next year >>> develop
into adults.
• Female worms become encapsulated in subQ nodules called
onchocercomas and are fertilized by males. They can remaine there for
up to 20 years. Location of nodules on body may depend on region in
which you were infected.
• Gravid females produce 1000’s of tissue invasive microfilariae each day >>> invade subQ tissues, skin and eyes.

During a blood meal, an infected blackfly (genus Simulium) introduces third-stage filarial larvae onto the skin of the human
host, where they penetrate into the bite wound . In subcutaneous tissues the larvae develop into adult filariae, which
commonly reside in nodules in subcutaneous connective tissues . Adults can live in the nodules for approximately 15
years. Some nodules may contain numerous male and female worms. Females measure 33 to 50 cm in length and 270 to
400 μm in diameter, while males measure 19 to 42 mm by 130 to 210 μm. In the subcutaneous nodules, the female worms
are capable of producing microfilariae for approximately 9 years. The microfilariae, measuring 220 to 360 µm by 5 to 9
µm and unsheathed, have a life span that may reach 2 years. They are occasionally found in peripheral blood, urine, and
sputum but are typically found in the skin and in the lymphatics of connective tissues . A blackfly ingests the
microfilariae during a blood meal . After ingestion, the microfilariae migrate from the blackfly's midgut through the
hemocoel to the thoracic muscles . There the microfilariae develop into first-stage larvae and subsequently into third-
stage infective larvae . The third-stage infective larvae migrate to the blackfly's proboscis and can infect another
human when the fly takes a blood meal .

17. What are the clinical manifestations of Onchocerciasis?


• Earliest manifestation is itching >>> dermatitis with flattened papules. Can
be so severe that patients commit suicide!
• Painless, moveable nodules due to onchocercomas
• Atrophy is relatively common in areas of high endemnicity >>> atrophic skin
looks like aged skin with wrinkling, loss of elacticity, loss of hair.
1. loss of elasticity can cause “hanging groin” that looks like hernia
• Depigmentation can occur >>> “leopard skin”
• Nodules where the female resides may be palpable over bony prominences.
• Vision impairment, especially in males >20; possible complications with
blindness as potential sequelue:
1. eye involvement
2. punctuate keratitis
3. anterior uveitis
4. Chorioretinal disease
5. Optic neuritis
Look for high IgE levels in all Filarial infections.

18. How is Onchocerciasis diagnosed?


• Skin snip commonly used for diagnosis >>> biopsy is placed in saline and observed for emergence of
microfilariae.
• Can also be done by surgical removal of subQ nodules or aspiration of nodules.
• A slit lamp can be used to examine eyes and demonstrate intraocular microfilariae.
• Serology via PCR is possible if skin snip is negative.
• Mazotti Test: give patient DEC >>> mobilizes worms >>> leads to itching >>> diagnostic of disease.

Section of onchocerca nodule.

19. How is Onchocerciasis treated?


• Ivermectin (similar to Strongyloides)

20. What is Bancroftian Filariasis?


• Bancroftian filariasis is caused by mosquito-borne nematode
Wuchereria bancrofti & Brugia Malayi.
• Adult worms live in lymphatic vessels and nodes of humans.
Females produce first stage larvae called microfilariae there >>>
they then circulate in blood.
• Lymphatic drainage is caused by local damage by the highly
motile adult worms.
• With progressive infection, inflammatory immune damage to
lymphatics results in a wide spectrum of clinical manifestations
that depend on part affected.
• Humans are the only natural host for Wuchereria bancrofti.
• Adults can remain in humans for many years.

21. What is the morphology of Wuchereria bancrofti?


• Adult female W. bancrofti found in lymph nodes and lymphatic channels are 10 cm x 250 micrometers whereas
males are only half that size.
• Microfilaria found in blood are only 260 micrometers x 10 micrometers.

22. What is the life cycle of W. bancrofti?


• An infected mosquito feeds on host (at night) and infective larvae enter host’s skin.
• The larvae develop into adults within lymphatic channels.
• Microfilariae have nocturnal variation: they migrate to lungs during the day b/c of higher PO2, then at night
they migrate to the bloodstream b/c PO2 highest there at night. If patient works at night, cycle will be reversed.
• Mosquitos ingest microfilariae from host’s blood.

Different species of the following genera of mosquitoes are vectors of W. bancrofti filariasis depending on geographical
distribution. Among them are: Culex (C. annulirostris, C. bitaeniorhynchus, C. quinquefasciatus, and C. pipiens);
Anopheles (A. arabinensis, A. bancroftii, A. farauti, A. funestus, A. gambiae, A. koliensis, A. melas, A. merus, A.
punctulatus and A. wellcomei); Aedes (A. aegypti, A. aquasalis, A. bellator, A. cooki, A. darlingi, A. kochi, A. polynesiensis,
A. pseudoscutellaris, A. rotumae, A. scapularis, and A. vigilax); Mansonia (M. pseudotitillans, M. uniformis); Coquillettidia
(C. juxtamansonia). During a blood meal, an infected mosquito introduces third-stage filarial larvae onto the skin of the
human host, where they penetrate into the bite wound . They develop in adults that commonly reside in the lymphatics
. The female worms measure 80 to 100 mm in length and 0.24 to 0.30 mm in diameter, while the males measure about
40 mm by .1 mm. Adults produce microfilariae measuring 244 to 296 μm by 7.5 to 10 μm, which are sheathed and have
nocturnal periodicity, except the South Pacific microfilariae which have the absence of marked periodicity. The
microfilariae migrate into lymph and blood channels moving actively through lymph and blood . A mosquito ingests the
microfilariae during a blood meal . After ingestion, the microfilariae lose their sheaths and some of them work their way
through the wall of the proventriculus and cardiac portion of the mosquito's midgut and reach the thoracic muscles .
There the microfilariae develop into first-stage larvae and subsequently into third-stage infective larvae . The third-
stage infective larvae migrate through the hemocoel to the mosquito's prosbocis and can infect another human when the
mosquito takes a blood meal .
23. What are the clinical manifestations of Bancroftian Filariasis?
• Signs and symptoms of Bancroftian filariasis vary widely form one endemic area to another. Differences may be
due to relative intensities of different vectors in different regions.
• Asymptomatic individuals are seen in endemic areas.
• Early on in disease, patients will experience acute attacks of retrograde lymphangitis, accompanied by fever,
chills, and malaise lasting 3-15 days. Can occur several times per year.
• Patients give history of: pain, erythema, tenderness in area of affected lymphnodes for hours or 1 day prior to
onset of lymphangitis.
• Acute filariasis: microfilariae in blood, eosinophilia, and increased IgE. Some individuals may only have a few
attacks in a lifetime. Months to years of acute episodes ranging from very mild to severe are followed by
development of chronic obstructive disease due to lymphatic insufficiency.
• Chronic filariasis: recurrent episodes of lymphedema of the extremeties will progress to elephantiasis (Focal
Point of O’Connor: point of lymphagenic spread in scrotum). The lower extremities are more commonly
affected. In many areas, most common chronic manifestation is a hydrocele. As a result of blockage of draining
lymph, straw colored hydrocele fluid accumulates (sometimes in huge volumes). Can have edema in any region
containing lymphatics.
1. Elephantiasis
2. NO IgE & NO Eosinophilia
3. Chyluria: lymphatic varices in bladder burst >>> lymphatic fluid in bladder & urine.
Tropical Pulmonary Eosinophilia: is an allergic reaction to microfilaria (Wuchereria bancrofti) which occurs
particularly in the Asian subcontinent (also Eastern Europe?). Presentation is with cough & asthma, fever,
lassitude, and weight loss. Chest X-ray shows bilateral hazy mottling that is equally distributed throughout both
lung fields. This condition is commonly accompanied by false positive serological tests for syphilis. There are
high titres of cold agglutinins. Treat w/ DEC.

24. How is Bancroftian Filariasis diagnosed?


• The presence of microfilariae on blood smear.
• Serology is available.

25. How is Bancroftian Filariasis treated?


• DEC
• Albendazole and Ivermectin are being studied for this use.

Image: Lymph node containing adult Wuchereria.

LECTURE #8: TRICHINOSIS & AMEBIASIS

Trichinosis
• Trichinosis caused by Trichinella and is acquired by eating muscles of wild or domestic animals.
• Severity is related to number of larvae ingested, and characterized by:
1. fever
2. GI symptoms
3. myositis
4. swollen eyelids
5. Eosinophilia - very high
• Transmission: through eating pork, bear, or horse meat
• Number of infections is declining. General drop in reported cases, but
probably milder so more cases than reported.
• Zoonosis: swine, rats
• Distribution: Occurs all over the US and the world.

Life Cycle of Trichinella Spiralis


• Eat undercooked pork, bear, cougar, horse, walrus meat >>> cysts (resistant
to stomach pH) exit muscle mass of meat >>> migrate to person’s small
intestine >>> burrow in tissue above lamina propria and undergo 4 molts in
36 hours >>> mature into adult males and females >>> copulation occurs >>> female gives birth to newborn
larvae within 4-7 days (viviparous event: female gives birth to live larvae, no eggs) >>> larvae travel through
bloodstream and lymphatics >>> go to person’s muscle masses (can pass through any striated muscle, but do not
encyst in cardiac muscle, only striated muscle) form nurse cell >>> then they coil around day 17 >>> encyst 2-3
weeks post-infection and within 6 months -1 year calcify >>> if you eat that calcified cyst you get sick.

• Any muscle mass can get involved: masseters, exta-ocular, nuccal, intercostals, gluteus, biceps, triceps, lower
extremities
• Larvae can go through CNS and myocardium, but won’t encyst there
• Nurse cell larva complex
• Trichinella pseudospiralis: does not form cysts

Clinical Presentation of Trichinosis


• Common clinical findings:
1. enteral stage: nausea, vomiting, diarrhea – looks like food poisoning
2. parental stage: weakness, myalgia, fever, periorbital edema, conjunctivitis and conjuctival
hemorrhages, hemorrhages in nail beds
• uncommon complications:
1. myocarditis: don’t encyst there, but do pass through the heart and can
rarely cause arrhythmias, EKG changes, and death
2. vasculitis
3. meningoencephalitis
• Even after calcification can still cause
disease

Images: splinter hemorrhages and periorbital


swelling seen with Trichonosis.

Laboratory Findings
• High eosinphilia (40-60% typically)
• Leukocytosis
• Elevated muscle enzymes (CPK, Aldolase, SGOT)
• Elevated IgE
• Positive serology: shows up 2-3 weeks later

Degenerating T. spiralis in muscle with calcium deposition.

Treatment
• Mebendazole
• Albendazole
• Corticosteroids for severe disease: if neuro or myocarditis, problem is that people think you might be interfering
with natural immunological response of the host, but this is only a theoretical problem

Prevention
• Cook meat to 160 degrees F is best
• Banning of feeding garbage including animal carcasses to swine and other barnyard animals.

Amebiasis
• Amebiasis: Should always be in your differential diagnosis, seen frequently in the US (4%), also very common in
Mexico
• Worldwide distribution, due to unclean water and poor sanitation, 10% of world infected, and up to 50% in some
developing countries.
• Causes a fair amount of mortality

Entamoeba Dispar vs. E. Histolytica


• Entamoeba Dipsar is morphologically identical to Entamoeba histolytica, but completely different effects on
human infection.
• E. Dispar is more common and solely asymptomatic carrier state, whereas pathogenic species E. histolytica can
invade tissue and cause symptomatic disease.
• Galactose-inhibitable adherence lectin is absent on Entamoeba dispar (asymptomatic) so can’t invade the host,
Entamoeba histolytica (symptomatic) can invade and cause symptoms and mortality.
1. important adhesion antigen
2. highly conserved
3. monoclonal probes differentiate the two, otherwise, morphologically indistinguishable
• First suggested in studies with zymodemes through electrophoresis
• Transmission: oral-fecal
• Allergies, malignancies, rheumatic diseases and parasites >>> high level of eosinophilia

Left image: Entamoeba histolytica trophozoite, note: fine uniform granules of peripheral chromatin and small central
karyosome in nucleus, size over 12 uM. Right image: Entamoeba histolytica trophozoite (8-30µm), note: ingested red
blood cells.

High Risk Areas


• Mexico, Columbia, Ecuador, Chile, Peru
• West Africa
• South Africa
• Middle East
• South and southeast Asia
• In part, based on poorer sanitation and nutrition.
• Lows socioeconomic status, crowding, poor nutrition, poor sanitation/plumbing, institutionalized populated
(mentally ill, or elderly), communal living, sexual behavior

Life Cycle
• E. histolytica is an enteric protozoa that exists as either trophozoite or cyst.
• Infection acquired from ingestion of cysts (quadranucleate and resistive to low stomach pH) from environment
>>> excysts in intestine >>> forms 8 trophozoites >>> invade large intestine feeds on bacteria and cellular debris.
• Trophozoites may then encyst
• Upon excretion, cysts remain viable for weeks to months depending on environmental conditions.
• Cyst infectious, trophozoite invades
Cysts and trophozoites are passed in feces . Cysts are typically found in formed stool, whereas trophozoites are typically
found in diarrheal stool. Infection by Entamoeba histolytica occurs by ingestion of mature cysts in fecally contaminated
food, water, or hands. Excystation occurs in the small intestine and trophozoites are released, which migrate to the
large intestine. The trophozoites multiply by binary fission and produce cysts , and both stages are passed in the feces
. Because of the protection conferred by their walls, the cysts can survive days to weeks in the external environment and
are responsible for transmission. Trophozoites passed in the stool are rapidly destroyed once outside the body, and if
ingested would not survive exposure to the gastric environment. In many cases, the trophozoites remain confined to the
intestinal lumen ( : noninvasive infection) of individuals who are asymptomatic carriers, passing cysts in their stool. In
some patients the trophozoites invade the intestinal mucosa ( : intestinal disease), or, through the bloodstream,
extraintestinal sites such as the liver, brain, and lungs ( : extraintestinal disease), with resultant pathologic manifestations.
It has been established that the invasive and noninvasive forms represent two separate species, respectively E. histolytica
and E. dispar. These two species are morphologically indistinguishable unless E. histolytica is observed with ingested red
blood cells (erythrophagocystosis). Transmission can also occur through exposure to fecal matter during sexual contact (in
which case not only cysts, but also trophozoites could prove infective).

Cysts
• Never see cysts in tissue, only trophozoites
• Very hardy
• Killed by boiling over 100 degrees Celsius, lives in ice
• Resistant to chlorine
• Survives for days on salads, cheeses and yogurt (20-25 degrees)
• In the stool, you see round cysts with peripheral chromatin with thick hyaline cell wall that looks like a halo (not
able to differentiate from ED).
• Stacking of ribosomal RNA used as an apparatus for cell division

Entamoeba histolytica cyst demonstrating chromatidal bars and 4 nuclei.

Trophozooite
• Pathogenesis: lysis
• Non-inflammatory disease
• Amoeboid trophozoite nucleus with peripheral chromatin
• Can invade, and on contact can lyse host tissue or cause host apoptosis, can ingest debris in intestine and RBCs
and WBCs due to lectin (wouldn’t see RBCs in E. dispar).
• Unidirectional motion of trophozoite

Mortality and Amebiasis


• High mortality in pregnant women, children <1 year, malnourished, and patients on corticosteroids.
• Can be mistaken for IBD >> treated with corticosteroids >>> potential development of Toxic Megacolon
requiring colectomy.

Pathogenesis of Amebiasis
• Initial lesion is cecal and then extends down to rectum.
• Can get extra-intestinal disease
1. hepatic: pretty common
2. pleuropulmonary
3. pericarditis
4. cerebral
5. other
• adherence of trophozoites to the mucus lining of large bowel – vital role in pathogenesis
• adhere to colonic mucins, epithelial cells and leukocytes >>> Galactose-inhibitable adherence lectin has
difficulty adhering but easier if malnourished have thinner layer of protective mucin.
• Bantu tribe lives primarily on corn, poor nutritional status >>> higher infection rates
• If in bowel can get into blood vessels and then disseminate into portal blood vessels and travel to liver
• Contact-dependent cytolysis: EM of cells pre and post infection of EH. After, see autolysis >>> release of
enzymes >>> causing destruction of local tissue. So organism invades by causing host cells to autolyse >>> then
organism can move easily through dead tissue.
• Phagocytoizes PMNS and other cells >>> release of lysosomal agents >>> more necrosis
• Foci of necrosis coalesce to form classic flask-shaped amebic ulcers.
• Deeper tissue invasion >>> vasculitis and thrombosis >>> transmural disease and tissue death
• Virulence: depends both on amoeba and host factors (someone who is healthy and gets low dose of pathogen less
likely to develop disease). Nutritional status very important: immunocompromised, corticosteroids, malignancies,
and malnutrition predispose to infection.
Entamoeba histolytica infection of rectum, note: ulcerative lesion, usually described as flask shaped ulcer.

Intestinal Disease
• Asymptomatic: majority pass cysts in stool with no symptoms, but cysts are infectious so it is a public health
issue. Carries a low, but definite possibility of invasion.
• Mucosal Disease:
1. acute rectocolitis (dysentery): bloody diarrhea and tenesmus
2. chronic non-dysenteric colitis: person has diarrhea for months on and off, can look like
inflammatory bowel disease (AI disease of colon presents with bloody diarrhea that lasts for months)
• Transmural Disease:
1. fulminant colitis with perforation due to coalescence of ulcers >>> high M&M
2. Toxic Megacolon: much more common in children, very bad complication due to transmural
disease and thinning of the bowel >>> huge cecum (may require a cecostomy otherwise will burst
and life-threatening illness).
• Ulcerative post-dysenteric colitis

Amoebic Colitis
• Does not have rapid onset like Salmonella and Shigella (normally associated with high fever).
• Gradual onset of 1-3 weeks, weight loss common due to gradual onset
• Abdominal pain and tenderness
• Bloody stools and tenesmus
• Fever 1/3 patients
• Path: flask-shaped ulcers with necrotic debris in the middle, on the edge trophozoites
• Differential diagnosis:
1. Infectious: amebiasis, Shigella, Salmonella, Campylobacter, Enteroinvasive and Enterohemorrhagic
E. coli
2. Non-infectious: ischemic colitis, AV malformation, Diverticulitis, inflammatory bowel disease
• Treatment for IBD is steroids. Must consider amoebic colitis, b/c if patient has that and you misdiagnose and give
steroids you will make patient sicker.
• Amoebic serology directed at 170 kD lectin will become positive with 1-2 weeks of infection, quite specific, but
can remain positive for up to 10 years.
• Complications:
1. appendicitis
2. perforation & peritonitis
3. ameboma: pain and tenderness, can present as an apple-core lesion on CT, usually secondary to
malignancy
4. amebic stricture: fibrosis secondary to healing transmural disease
5. cutaneous amebiasis

Treatment
• Tissue-cidal: Flagyl=Metronidazole (not good for cysts, but good for invasive disease)
• Luminicidal: Iodoquinol, Paromomycin, Diloxanide Furoate (in other countries), good for asymptomatic cyst
characteristics

Differential Diagnosis for Bloody Diarrhea


• Amoebiasis
• Shigallosis
• Campelobacter jejuni
• E. coli (Enteroinvasive, Enterohemorrhagic)
• Clostridium difficile
• Testing: stool for O&P, Assay for toxin of C. difficile, and
• Treat: with Flagyl, follow up with Luminicidal agent

Conclusion
• Dispar never invasive >>> don’t need to treat
• Cyst doesn’t invade (it is infective stage); trophozoite is invasive stage
• Amoebiasis: ring enhancing lesion in brain, space-occupying cystic lesions can develop in brain, lung, or liver.

Hepatic Amebiasis
• Hepatic infection results from trophozoites ascending portal venous system.
• Predilection for right lobe, usually singular, but can be multiple. Trophozoites cause cells to self destruct >>> get
a lot of necrotic debris. Also, lysis of PMNs >>> release of destructive host enzymes. Only see Trophs in edge of
lesion. This is called a “liver abscess,” but misnomer (no pus, not an abscess). If grows can get elevation of
hemidiaphragm. Can rupture into lung or peritoneum. If on right can rupture into heart.
• If you put a needle into an amoebic abscess, you see Anchovy paste or chocolate-like paste, vs. regular abscess
see sheets of PMNs.
• Males are 10x more likely to get invasive Amebiasis than females.
• Patient presentation: RUQ tenderness and fever. Patients look really ill and toxic. (In amoebic colitis many don’t
have fever). On chest x-ray see elevated right hemi-diaphragm >>> something is going on in right sub-phrenic
space. Typically seen in patients who have traveled to endemic location. Normally, not acute onset (4 weeks). A
history of diarrhea is only present in 1/3 of patients, vs. amebic colitis where patients have bloody diarrhea.
Cough present in 1/3 of patients.
• See + amebic serology, but can remain + for up to 10 years.
• Path: collapse of stroma of liver, with necrotic debris. See invasive trophozoites on edge of lesion.

Differential Diagnosis of Space Occupying Lesion & Fever


• Differential diagnosis of space occupying lesion and fever >>> bacterial abscess (older patient 50+), diabetics,
underlying disease such as cancer or biliary disease, men=females), Hepatic Amebiasis (travel, younger patient,
men>females), tumors (usually lower grade fever), Echinococcus (dog tapeworm) morphology looks different, or
Fasciola Hepatica.
• Put a needle into abscess if think its pyogenic abscess, or if a liver abscess is >5cm or it looks as if it’s going to
perforate.
• Treatment: Flagyl + luminicidal agent such as Iodquinol

Lab Tests
• Leukocytosis (12,000-20,000 mm^3)
• ESR is elevated (non-specific marker of inflammation)
• Normocytic-normochromic anemia
• Alkaline phosphatase elevated
• Partial immunity develops

LECTURE #9: DIENTAMOEBIASIS, GIARDIASIS & TRICHOMONIASIS

General Notes
• Most intestinal protozoa have cyst >>> excyst in intestine >>> amplify in intestinal lumen >>> trophozoites
invade >>> trophozoites can then encyst and go out into environment.

What is Dientamoebiasis?
• Dientamoebiasis is caused by Dientomoeba fragilis, an ameba-like flagellate closely related to the genera
Histomonas and Trichomonas.
• D. fragilis has a worldwide distribution
• Transmission: unknown. May be water borne and person-to-person transmission. Close relationship between
Enterobius vermicularis and D. fragilis infections.
• Non-invasive in large intestine.
• It doesn’t have cyst phase, and it is a trophozoite >>> not sure how it gets around. It has been postulated that
pinworm eggs or larvae may be transmitting agent for D. fragilis.

What is the pathogenesis of Dientamoebiasis?


• Does NOT seem to invade. Theory of transmission is that this protozoa gets from person-to-person via the
pinworm eggs. So if patient diagnosed with either pinworm or Dientamoebiasis – look for the other parasite as
well!
• D. fragilis causes symptoms by irritating colonic mucosa.
• Fibrosis is present in appendices infected by D. fragilis.

What are the clinical manifestations of Dientamoebiasis?


• Many asymptomatic
• Can present with: intermittent diarrhea, abdominal pain, anorexia, fatigue, and less commonly with fever,
irritability, weight loss and vomiting.
• Occasionally see eosinophilia b/c of pinworm.

How is Dientamoebiasis diagnosed?


• Diagnosis by finding trophozoites in stool.

How is Dientamoebiasis treated?


• Tetracycline & Iodoquinol, alone or in combo
• Paromomycine can also be used

What is Trichomoniasis?
• It is a specific GU tract infection with Trichomonas vaginalis.
• The organism is highly site specific >>> vaginitis in women and urethritis in men (both sexes are equally
susceptible, but women symptomatic and men tend not to be, so treat both men and women, if woman presents).
• Distribution: occurs worldwide, in both rural and urban settings.
• Transmission: it is a common STD and patients should be screened for other STDs if positive. Perinatal
transmission occurs rarely in vaginal births. No cyst forms! Transmission requires intimate contact with actively
motile organism. So, if find in a child, be concerned about child abuse.
• At risk groups: professional sex workers, young people who are more sexually active

Trichomonas vaginalis typically infects the vagina in women and the prostate and urethra in males.

What is the morphology of Trichomonas vaginalis?


• 3-5 anterior flagella
• One undulating membrane
• Hydrogensome
• 7-23 micrometers
• Axostyle

What is the life cycle of Trichomonas?

Trichomonas vaginalis resides in the female lower genital tract and the male urethra and prostate , where it replicates by
binary fission . The parasite does not appear to have a cyst form, and does not survive well in the external environment.
Trichomonas vaginalis is transmitted among humans, its only known host, primarily by sexual intercourse .

What is the pathogenesis of Trichomoniasis?


• Mechanisms of this disease poorly understood.
• Organism can be found in vagina of >95% of infected women and urinary tract alone in <5%.
• Infects squamous epithelium, but not columnar epithelium, and organisms are rarely isolated from endocervix.
• Urethra involved in 90% of cases, and organisms can be isolated from urine.
• Can be found in men in epididymis and prostate.
• Association with cervical carcinoma due to co-infection with Papillomavirus.

8. What are the clinical manifestations of Trichomoniasis?


• Trichomoniasis ranges in women from asymptomatic to severe inflammation. Other symptoms:
1. vaginal discharge (often yellow due to lysed PMNs)
2. vulvar irritation or pruritis
3. dyspareunia
4. lower abdominal discomfort w/ adnexal tenderness on bimanual exam
5. sometimes strawberry cervix
6. vaginal wall erythema
7. the organism is cytolytic, and pumps out a lot of polyamines >>> fishy odor w/ copious discharge.
8. pain on urination
9. often exacerbated by menstruation
• Initially asymptomatic disease frequently becomes symptomatic >>> so asymptomatic carriers should be treated.
• Disease usually asymptomatic in men, but can rarely present as urethritis, epidiymitis or prostate involvement.
• Many patients co-infected with gonorrhea or other STDs.

What are some potential complications of Trichomoniasis?


• Usually benign
• But in pregnant women can cause premature rupture of membranes >>> early childbirth and fetal complications
• May be associated with enhanced sexual transmission of HIV (seroconversion), especially in developing
countries.

How is Trichomoniasis diagnosed?


• Wet mount preparation of vaginal fluid
• Urine sediment
• Prostatic secretion
• pH > 4.5
• Fluorescent antibody test is most specific and sensitive
• Culture >95%, but rarely done b/c takes 3 days and patients present with acute symptoms
• Positive whiff test

How is Trichomoniasis treated?


• Flagyl/Metronidazole: can use single dose therapy
• Simultaneous treatment of partner!
• 90% cure rate – recurrent symptoms do not necessarily indicate resistance to Metronidazole, more likely to be
reinfection.
• Prevention: using condoms, limiting number of sexual partners

What is Giardiasis?
• A zoonotic infection (but most commonly person-to-person transmission) of the lumen of the small intestine with
the flagellate protozoan Giardia lamblia.
• Many patients have no symptoms (50% spontaneous cure w/o symptoms), but a smaller portion have diarrheal
disease that varies in its severity (can be acute or chronic diarrhea, usually 1+ week after exposure).
1. usually non-bloody, explosive diarrhea (it is an upper intestinal parasite, that is NOT invasive,
prevents absorption).
• Distribution: occurs worldwide in temperate and tropical areas. Essentially a disease of poverty where sanitation
is poor, fecal contamination of environment is common, and there is no clean drinking water. 200 million cases
annually.
• Transmission: infection follows ingestion of viable parasitic cysts from contaminated food or water. Cysts can
also be transmitted via fingers to mouth particularly in children (after touching ground contaminated with feces).
Person-to-person transmission also occurs in children.
1. US outbreaks: defective water treatment plants
2. beavers and dogs
3. tropics >>> high level of environmental contamination
4. can get it from drinking from mountain streams
• Symptomatic disease more common in children
• Mechanism of diarrhea with Giardia is unknown. Giardia contains viruses (mini Rotovirus) – may be involved in
diarrhea, but doesn’t seem to be.

What is the morphology of Giardia lamblia?


• Cyst is oval and measures 8x12x8 micrometers
• Can see median body and 4 nuclei
• Pear-shaped trophozoite is 15 micrometers long, with curved dorsal surface and a concave ventral surface, much
of which is taken up by a disk with regular striations termed the sucker disk.
• Has no mitochondria, but does have some human or mammalian genes in Giardia genome. Entameba histolytica,
Trichomonas, etc also lack mitochondria >>> treat with Metronidazole b/c anaerobic.
• 2 of 8 flagella emerge near midline on the ventral surface of organism, and others around periphery (3 on each
side) >>> the beating flagella produce spiraling motion and keep the organism attached to microvilli of gut
epithelium

Left image: Giardia lamblia cyst (8-12 long x 7-10µm wide) with 2 nuclei. The mature cyst has 4 nuclei. Right image:
Giardia lamblia trophozoite (9-21 long x 5-15µm wide). Typically found in the duodenum. It is binucleate and has 4 pairs
of flagella. Giardia is a Diploid organism with 2 1N nuclei.

What is the life cycle of Giardia lamblia?


Cysts are resistant forms and are responsible for transmission of giardiasis. Both cysts and trophozoites can be found in the
feces (diagnostic stages) . The cysts are hardy and can survive several months in cold water. Infection occurs by the
ingestion of cysts in contaminated water, food, or by the fecal-oral route (hands or fomites) . In the small intestine,
excystation releases trophozoites (each cyst produces two trophozoites) . Trophozoites multiply by longitudinal binary
fission, remaining in the lumen of the proximal small bowel where they can be free or attached to the mucosa by a ventral
sucking disk . Encystation occurs as the parasites transit toward the colon. The cyst is the stage found most commonly
in nondiarrheal feces . Because the cysts are infectious when passed in the stool or shortly afterward, person-to-person
transmission is possible. While animals are infected with Giardia, their importance as a reservoir is unclear.

What is the pathogenesis of Giardiasis?


• Infected individuals have a range of functional and morphological changes.
• Those who are asymptomatic usually have normal absorption and normal jejunal mucosa.
• Blunting of villi of brush border in small intestine >>> poor absorption.
• Mild symptoms associated with some reduction in D-xylose absorption and minor histologic abnormalities without
reduction of villous height.
• Impaired absorption of fat, D-xylose and Vitamin B12 are common in those severely affected. There is obvious
reduction in villous height, increase in crypt depth with increased numbers of mitotic figures.
• Enzyme deficiencies >>> lactose intolerance.
Giardia lamblia on intestinal biopsy.

What are the clinical manifestations of Giardiasis?


• Symptoms in acute infections begin after an incubation period of 1-2 weeks, then resolve spontaneously, and
include:
1. anorexia
2. nausea
3. lassitude
4. diarrhea w/ yellow, frothy, greasy, foul-smelling stools (no blood or mucous)
5. bad taste in mouth
6. sulfuric flatulence and belching
7. abdominal discomfort
8. weight loss
9. steatorrhea
• after infection a variety of outcomes:
1. acute diarrheal illness that resolves spontaneously with eradication of infection
2. continuing diarrhea and weight loss with continuing infection
3. resolution of diarrhea with continuing cyst excretions
• complications:
1. if symptoms present for months >>> glottitis, indicating folate deficiency
2. food allergy

How is Giardiasis diagnosed?


• Demonstration of parasite in stool samples or path specimens (low vs. high excretors), take 3 stool samples (>95%
probability of finding organism, but can be same stool 3 times)
1. can run monoclonal Antibodies on stool sample
2. Copro-antigens (antigens in stool) is an ELISA test, but high false positive rate.
• ELISA and monoclonal antibodies have both been used with good results.
• Enterotest: string with lead weight, swallow capsule, it dissolves in stomach, look for bile at end of capsule and
get a sample of upper intestinal contents. Low-tech.
• Endoscopy and biopsy
• serology

Who gets Giardia?


• People with poor hygiene, such as children and food workers
• Backpackers or travelers
• People in areas with poor sanitation
• Male homosexuals
• Water-borne epidemics

How is Giardiasis treated?


• Metronidazole
• Tinidazole
• Quinacrine
• Furazolidone
• Paromomycin: if patient is pregnant
• Usually not resistant, typically re-infection, or person is immunocompromised
• Prevention: personal hygiene and prevention of fecal-oral transmission, protect water supply >>> boiling will get
rid or most pathogens, iodine is effective, chlorine is less effective

Balantidium coli
• B. coli is a classic ciliate
• Invasive >>> bloody diarrhea
• Covered with rows of cilia
• Defining characteristic is nuclear dimorphism
• Sexual reproduction involving conjugation
• Primarily seen in people with exposure to pigs and other
animals.
• Treatment: Tetracycline or Iodoquinol
• Diagnosis: look for cysts or trophozoites in stool

Balantidium coli, note: only ciliate infecting man, macro and


micronucleus present.

Blastocystis hominis
• Life cycle unknown
• Previously called yeast, ameba, sporozoan, etc. Appears to be a Stramenophile.
• Pathology: on the border between commensal organism and pathogen >>> can lead to vague intestinal discomfort.
Fecal-oral transmission.
• Treatment: no real treatment. Can use Iodoquinol or Metronidazole, but only 50-60% effective.

LECTURE #10: TOXOPLASMOSIS

Toxoplasmosis
• Major cause of M&M, caused by the protozoan parasite Toxoplasma gondii.
• Most infections are asymptomatic, with at most minor flu symptoms. Infections most severe in congenitally
infected infants and in immunocompromised.
• Transmission:
1. ingestion of tissue cysts containing bradyzoites in undercooked infected meat
2. ingestion of environmentally resistant sporulated oocytes in water, raw produce, or soil
3. transplacentally from mother to fetus
4. tissue transplantation
• Cats (particularly younger cats and kittens) and field animals are definitive hosts (adult forms and sexual
reproduction), and most warm-blooded animals can serve as intermediate hosts (immature forms and asexual
reproduction).
• 22.5% of people in US have antibodies to it – very common parasitic illness.
• Treatment: drugs can treat active infection, but none kill the tissue cyst stage of parasite.
• Oocysts are environmentally stable can survive in water, etc.
• Latent tissue cyst stage parasites can reactivate and cause development of new illness in immunocompromised
patients.
• Distribution: worldwide, widespread infections in mammals and birds.
Toxoplasma gondii stages; A) bradyzoite, B) tachyzoite, C) tachyzoites in tissue culture, D) oocyst [Nomarski optics].

Apicoplasts
• A class of parasites
• Apicoplast: organelle with 4 membranes, typical plastid has 2 membranes as do mitochondria (potentially b/c of
joining of 2 cells). Potentially ingestion of a fungi.
• Toxoplasmosis and Malaria have several plant-like metabolic pathways >>> major potential therapeutic target!!
• Unique mechanism of replication called endogeny. Form 2 daughter cells within eukaryotes.

Life Cycle of Toxoplasma Gondii


A. Ingestion of infected meat
• When infected meat with tissue cysts is ingested, the bradyzoites are liberated from tissue cysts after exposure to
the acid conditions of the stomach >>> bradyzoites penetrate the mucosa of the SI >>> multiply in cells in lamina
propria.
• Bradyzoites convert to tachyzoites within a few days (rapidly reproducing form goes through reticuloendothelial
system).>>> tachyzoites disseminate throughout body via lymphatic and vascular systems >>> lymphadenopathy
and fever.
• Tissue cysts are then produced in various tissues such as heart, retina and skeletal muscles. They are viable for
several years to life of host.
Toxoplasma gondii cyst (bradyzoites) in lymph node biopsy.

B. Ingestion of Sporulated Oocysts


• When sporulated oocysts are ingested, the sporozoites are liberated from the sporocysts inside the oocysts into the
lumen of the duodenum. Sporozoites penetrate the mucosa of the SI >>> multiply >>> tachyzoites disseminated
throughout body >>> tissue cysts containing bradyzoites are produced in a variety of tissues.
C. Life Cycle in Cat
• T. gondii undergoes a coccidial life cycle in enterocytes of small intestine.
• 5 distinct asexual schizont types are present in enterocytes of feline intestine and produce the sexual stages that
give rise to oocysts.
• Oocysts are excreted in the feces for several weeks, but high numbers are only excreted during the first week of
patency. The length of time it takes before oocysts are excreted (prepatent period) varies depending on the stage
of T. gondii that was ingested by the cat. Oral inoculation of bradyzoites/tissue cysts is most efficient in inducing
oocyst-producing infections in cats. Oral inoculation with tachyzoites or oocysts is less efficient.
• Oocytes then develop (sporulate) in outside environment in a few days >>> the sporulated infective oocyst has 2
sporocysts that each enclose 4 infective sporozoites.
Members of the cat family (Felidae) are the only known definitive hosts for the sexual stages of T. gondii and thus are the
main reservoirs of infection. Cats become infected with T. gondii by carnivorism . After tissue cysts or oocysts are
ingested by the cat, viable organisms are released and invade epithelial cells of the small intestine where they undergo an
asexual followed by a sexual cycle and then form oocysts, which are excreted. The unsporulated oocyst takes 1 to 5 days
after excretion to sporulate (become infective). Although cats shed oocysts for only 1 to 2 weeks, large numbers may be
shed. Oocysts can survive in the environment for several months and are remarkably resistant to disinfectants, freezing,
and drying, but are killed by heating to 70°C for 10 minutes.
Human infection may be acquired in several ways: A) ingestion of undercooked infected meat containing Toxoplasma cysts
; B) ingestion of the oocyst from fecally contaminated hands or food ; C) organ transplantation or blood transfusion;
D) transplacental transmission; E) accidental inoculation of tachyzoites. The parasites form tissue cysts, most commonly in
skeletal muscle, myocardium, and brain; these cysts may remain throughout the life of the host

Pathogenesis of Toxoplasmosis
• Tachyzoites (proliferative stage) can be found in any cell type in the body except RBCs.
• Hundreds to thousands of Bradyzoites are contained in tissue cysts (latent stages), and can be found in many
tissues including brain, eye, heart, kidney, liver, bone marrow and skeletal muscles.
• Domestic cats excrete the largest amount of T. gondii sporulated oocytes in their feces >>> humans can become
infected after ingesting infected water, food, etc.
• Ingestion of unwashed fruits and vegetables is associated with increased risk of maternal toxoplasmosis, as is
contact with soil.
• Cockroaches can pick up cysts and deposit them other places
• Pork is the most likely commercial food sources of tissue cysts in the US. Chicken (rarely), sheep, goat, deer, wild
pigs, turkeys, and bear are all possible sources of infection – found in mammals. Can ingest zoites in undercooked
meat.
• Cats are pivotal in transmission of parasite to herbivores and omnivores
• Only the bradyzoite stage from the tissue cyst can cause the enteroepithelial cycle in the cat’s enterocytes, which
terminates in occyst excretion in feces.
• Sporulation occurs in environment and depends on temp and moisture.
• Organ transplantation can transfer infection if latent cysts or bradyzoites in tissue of transplanted organ.
• Blood transfusions can transmit T. gondii only during acute stage of infection.

Toxoplasma gondii in heart tissue (tachyzoites or pseudocyst).

Clinical Manifestations of Toxoplasmosis


• Symptoms depend on age and immune status of patient. Most cases asymptomatic.
• Infections in congenitally infected patients and immunocompromised are more severe.
1. CNS infection with encephalitis or chorioretinitis is most common manifestation.
• Acute disease and death can occur in a primary infection.
• Re-activated disease occurs in individuals who have a latent infection and subsequently become
immunocompromised.
• 1-2 week incubation period; peaks 1-2 months and then declines. IgG persists for life.
• In symptomatic patients:
1. mononucleosis-like symptoms with lymphadenitis or chorioretinitis most common
2. rarely, myocarditis, polymyositis, septicemia, or encephalitis
• Complications: depend on age and immune status of patient.
1. chronic behavioral changes – link made in literature, unknown
• T. gondii is important cause of chorioretinitis and posterior uveitis
• Congenital infections usually present by age 30, whereas acquired are
common age 40-70.
• Prior to HAART, most frequent cause of CNS lesions in HIV+ patient. It
is related to reactivation of latent infection. Need to biopsy brain for
diagnosis (cannot use serology). Or, can initiate therapy based on clinical
and radiographic syndrome with anti-Toxo therapy – if see response that
makes the diagnosis without brain biopsy.
Left image: Chorioretinits due to Toxoplasma gondii (Congenital infection). Right image: Encephalitis due to Toxoplasma
gonii, CT scan, in an AIDS patient. Note typical, but not diagnostic, ring enhancing lesion.

Congenital Infection with Toxoplasmosis


• Congenital infection occurs with women are seronegative during pregnancy
• Women who are seropositive before conception, do not transmit infection to fetus. Exception:
immunocompromised women (due to HIV or steroids) can transmit latent infection during pregnancy.
• Likelihood of transmission of infection is more likely with each progressive trimester, but severity of disease in
the fetus is worse the earlier in pregnancy it occurs.
• Symptoms of congenital infections:
1. chorioretinitis
2. strabismus
3. epilepsy
4. mental retardation
5. anemia
6. jaundice
7. rash
8. thrombocytopenia
9. encephalitis
10. microcephaly
11. sensorineural hearing loss
12. intracranial calcifications
13. hydrocephalus
• part of TORCH syndrome.
• 2/3 asymptomatic at birth, but may develop chorioretinits or other symptoms later

Diagnosis of Toxoplasmosis
• detection of anti-T. gondii antibodies by serological tests
• detection of tachyzoites or T. gondii-specific DNA in body fluids or tissue sample
• parallel testing of specimens 4 weeks apart
• seroconversion of a 4 fold rise in IgG titer is diagnostic of acute infection.
• In polymyositis or myocarditis biopsy can be useful (so low avidity antibody is seen during acute infection).
• IgG persists for life. Titer avidity increases with time, so low avidity IgG more likely to be acute infection.
• IgM appears earlier and declines more rapidly than IgG (usually within a few months) >>> presence of IgM
suggests acute infection.
• Low IgG titers in first trimester with negative IgM are consistent with chronic infection
• Positive IgM titers require further evaluation. 60% with + IgM have chronic infection.
• Amnio and US of fetus are recommended for acute infection with PCR. Fetal blood sampling does not increase
diagnostic yield.

Treatment of Toxoplasmosis
• Asymptomatic or latent infection:
1. immunocompetent individuals w/ latent disease do not require treatment
2. AIDS patients can get encephalitis >>> treat with TMP-SMX
3. cardiac transplantation treat prophylactically for 6 weeks with Pyrimethamine
• Acquired Toxoplasmosis:
1. if immunocompetent treatment probably not necessary
2. in cases of myocarditis, encephalitis, or sepsis, treat with Pyrimethamine + Sulfadiazine (or
Trisulfapyrimidine) + Folinic Acid (for 4-6 weeks)
• Acquired infection during pregnancy:
1. acutely infected pregnant women treat with Spiramycin
2. Amniocentesis, fetal blood monitoring and fetal US should also be used
3. If + fetal diagnosis >>> treat mother with Pyrimethamine + Sufladiazine + Folinic Acid. However,
Pyrimethamine should not be used during first 16 weeks of pregnancy.
• Congential Toxoplasmosis:
1. treat with Pyrimethamine + Sulfadiazine + Folinic Acid
2. corticosteroids should be added in patients with macular disease
• Toxoplasmosis infection in immunocompromised host:
1. Pyr + Sulfa + FA (Clindamycin can be substituted for Sulfa drugs)
2. Corticosteroids can be used to control intracranial HTN
• Alternative meds: Atovaquone, Azithromycin, Clindamycin, gamm-interferon (critical for almost all
intracellular parasites) recombinant form is experimental therapy, plant fatty acid pathway inhibitors like
Triclosan or Roundup

Prevention of Toxoplasmosis
• Don’t eat raw or poorly cooked meat
• Food preparers should wash hands and cooking surfaces with warm soapy water
• Change cats’ litter boxes regularly, but pregnant women and immunocompromised shouldn’t do this. Also, keep
cats indoors and feed only commercially prepared foods.
• Wear gloves when gardening
• Wash all fruits and vegetables

Pyrimethamine (PYR)/Daraprim
• Mechanism of Action: inhibits dihydrofolate reductase (Trimethoprim also inhibits this enzyme)
• Absorption: readily absorbed from GI tract
• Distribution: well distributed in most tissues, and found in CSF; t1/2= 35-175 hours
• Metabolism: hepatic
• Adverse Effects: dose-related bone marrow suppression w/ thrombocytopenia, neutropenia, and anemia. Folinic
Acid given to prevent these side effects.

Sulfonamides
• Mechanism of Action: inhibit dihydrofolic acid synthetase, which is another enzyme involved in folic acid
metabolism, so synergistic with PYR.
• Absorption: well-absorbed
• Distribution: good, with CSF
• Metabolism:
• Adverse Effects: are common (particularly in AIDS patients), bone marrow suppression (responsive to Folinic
Acid), hypersensitivity reactions with rash and Stevens-Johnson syndrome.

Macrolides & Lincomycins


• Drugs: Spiramycin (SPR, available in Europe and on request from FDA in US), Erythromycin, Azithromycin,
Roxithromycin, Clarithromycin
• Mechanism of Action: inhibition of apicoplast

Pneumocystis Carinii
• Originally considered a protozoa, but it is actually an atypical fungus.
• However, doesn’t respond to anti-fungal drugs. Responds to Bactrim (TMP-SMX) and other anti-protozoal drugs.
• Host specific organism (rats and humans have 2 different species).
• Pneumocystis jirovecii is the species that infects humans, so P. carinii is not actually the infective agent in PCP
seen in AIDS patients.
• Symptoms: primarily infects the lungs >>> pneumonia, SOB, chronic cough, low-grade or no fever, decreased
PaO2 on room air, chest tightness or pain.
• Who does it affect?
1. Primarily disease of immunocompromised (AIDS CD4 >200) and lethal if untreated. Occurs at less
immunosuppression than other op. infections >>> bellweather of HIV epidemic.
2. Malnourished infants
3. Children with primary malignancies
4. solid tumor patients, neoplasms, hematologic malignancies
5. high dose corticosteroids !!!
6. transplant recipients
7. inflammatory or collagen-vascular diseases (esp. Wegener’s granulomatosis)

Pneumocystis carinii, Lung section stained with Gomori methenamine silver stain; cyst wall is stained black. Know this
slide!!! Classic pneumocystosis shown by silver stain. Know it isn’t yeast b/c no budding.

Diagnosis of Pneumocystis
• a non-cultivatable organism, but can look for it in sputum with special stains or monoclonal antibodies
• bronchopsy for deep lung specimen
• Gallium scan
• Diffuse, foamy infiltrate in the middle of the lung
• Life cycle is not completely figured out
• Person-to-person transmission
• PCP increasing in HIV+ people in developing countries
• HAART is decreasing incidence
• Evidence for transmission form symptomatic hosts; some
outbreaks in hospital settings >>> may want to isolate
pneumocystis patients

Image: Pneumocystis carinii, giemsa stain; trophozoites are stained


(intracystic bodies are visible).

Prevention & Treatment


• Immune reconstition – if CD4 count above 200, incidence drops dramatically
• Bactrim prophylaxis can prevent pneumocystis >>> so given to all patients with CD4 count <200.
• Primaquine & Clindamycin
• Pentamidine not used anymore
• Alternative to TMP-SMX that is widely used is Atovaquone.
• Adjunctive corticosteroids in antipneumocystis therapy

LECTURE #11: CRYPTOSPORIDIUM, CYCLOSPORA, ISOSPORA & MICROSPORA

Comparison of Cyclospora, Cryptosporidium & Isospora


Acid fast staining:
1. Cyclospora: variable
2. Cryptosporidium: yes
3. Isospora: yes
UV Autofluorescence: Cyclospora blue/green

Cryptosporidium Species
• Intracellular GI parasites originally thought to cause disease in animals.
• Current methods of water purification do not remove Cryptosporidia.
• No effective therapy for it.
• Most cases self-limited, but can be devastating in immunocompromised, and to children in developing countries.
Immunocompromised with deficient cell-mediated immunity are particularly vulnerable.
• Types: 10 species, but Cryptosporum parvum infects humans.
• Infection is a zoonosis with mainly bovine and human reservoirs.
• Transmission: humans get infection by ingesting oocysts, which are shed in stool of animals or humans either
through water, food, and excreted fully infective so can be passed from person to person! Hardy cysts resistant to
chlorine, UV radiation, but killed by freezing or heating.
• Epidemiology:
1. Endemic in developing countries >>> one of most common causes of persistent diarrhea among children.
2. In developed world, mainly in outbreaks, usually due to contaminated drinking water.
3. Antibodies in 30% of US population.
4. Milwaukee & Nevada outbreaks
5. Water outbreaks

Life Cycle of Cryptospordium


Life cycle of Cryptosporidium.
(from: Juranek DD. Cryptosporidiosis. In: Hunter’s Tropical Medicine, 8th edition. Strickland GT, Editor.)

Sporulated oocysts, containing 4 sporozoites, are excreted by the infected host through feces and possibly other routes such
as respiratory secretions . Transmission of Cryptosporidium parvum occurs mainly through contact with contaminated
water (e.g., drinking or recreational water). Occasionally food sources, such as chicken salad, may serve as vehicles for
transmission. Many outbreaks in the United States have occurred in waterparks, community swimming pools, and day care
centers. Zoonotic transmission of C. parvum occurs through exposure to infected animals or exposure to water
contaminated by feces of infected animals . Following ingestion (and possibly inhalation) by a suitable host ,
excystation occurs. The sporozoites are released and parasitize epithelial cells ( , ) of the gastrointestinal tract or
other tissues such as the respiratory tract. In these cells, the parasites undergo asexual multiplication (schizogony or
merogony) ( , , ) and then sexual multiplication (gametogony) producing microgamonts (male) and
macrogamonts (female) . Upon fertilization of the macrogamonts by the microgametes ( ), oocysts ( , ) develop
that sporulate in the infected host. Two different types of oocysts are produced, the thick-walled, which is commonly
excreted from the host , and the thin-walled oocyst , which is primarily involved in autoinfection. Oocysts are
infective upon excretion, thus permitting direct and immediate fecal-oral transmission.
Note that oocysts of Cyclospora cayetanensis, another important coccidian parasite, are unsporulated at the time of
excretion and do not become infective until sporulation is completed. Refer to the life cycle of Cyclospora cayentanensis
for further details.
Pathogenesis of Cryptospordiosis
• Infection with Cryptosporidium begins when the ingested oocyst releases sporozoites >>> attach to and invade
intestinal epithelial cells (particularly jejunum and terminal ileum) >>> minimal inflammatory infiltrate (IFN-
gamma) and blunting of villus (more pronounced in immunocompromised) >>> malabsorption.
• Mechanism of diarrhea is unclear.

Clinical Manifestations of Cryptosporidiosis


• Presentation variable, typically profuse, watery diarrhea w/ abdominal pain; sometimes asymptomatic.
• Sometimes: fever, malaise, nausea, vomiting and loss of appetite.
• Symptoms 5 days-2 weeks post-ingestion of oocysts; lasts 3-5 weeks.
• Increased risk of infection with CD4 count <200.
• Acute Diarrheal Illness: shedding of oocysts for 8-50 days post-resolution of clinical symptoms:
1. watery diarrhea (+/- mucous)
2. epigastric pain, nausea, vomiting
3. weight loss
4. low grade fever 50% immunocompetent
• Chronic Diarrheal Illness: occurs in AIDS (CD4 <100), hypogammaglobulinemia, or chemo patients. Similar
presentation, but lasts months to years, may be severe or life-threatening. Will resolve spontaneously in patients
with reversible immunosuppression.
1. diarrhea w/ fluid losses of >20 L/day possible
2. severe abdominal pain
3. anorexia and major weight loss
• Gallbladder & Biliary Disease:
1. cholangitis seen in AIDS patients CD4 <50 – poor prognosis
• no RBCs or WBCs in stool

Diagnosis of Cryptosporidiosis
• biopsy w/ demonstration of intracellular parasite
• Acid-fast Kinyoun test to identify oocysts in stool – gold standard

Treatment of Cryptosporidiosis
• Nitazoxanide in children and HIV patients. HIV patients also need HAART.
• Azithromycin

Isospora
• Distribution: South America, Mexico, Carribean
• Symptoms: watery diarrhea, flatulence, malaise, anorexia, low fever, weight loss, Charcot-Leyden crystals in
stool, peripheral eosinophilia (exception to rule that protozoa don’t cause eosinophilia!).
• Transmission: sporulates outside of host – probably doesn’t have person to person transmission.
• Diagnosis: check stool for O&P. Acid Fast!
• Treatment: Bactrim; prophylax patients with low CD4
counts.

Left image:
Isospora belli
oocyst (20-33µm
x10-19µm)
containing 2
sporoblasts.
Right image:
Isospora belli
acid fast staining.
Cyclopsora
• Cyclospora is a genus in the phylum Apicomplexa. It is related to Cryptosporidium, Isospora, Toxoplasma, and
Sarcocystis.
• Like other Apicomplexans, sporozoites have an apical complex composed of a polar ring, conoid, rhoptries, and
other coccidian structures.
• Morphology: 8-10 microns in diameter.
• Life cycle: not known. Thought that sexual and asexual development occurs in human host. Cyclospora oocysts
in fresh stool are NOT infectious; oocysts require days to weeks outside host in favorable environmental
conditions to sporulate and become infectious.
• Transmission: through food (often fresh produce like raspberries), and water (highly chlorine resistant), human-
to-human unlikely.
• Distribution: common in tropical and subtropical areas worldwide; endemic to Nepal, Haiti & Peru. Rainy season
May to October, and in US May to July. Marked seasonality of infection – but with differing patterns in
different regions.
• Pathogenesis: intracellular organism that parasitizes enterocytes of upper small bowel. Inflammatory cell
infiltrates in lamina propria sometimes. Pathological changes are similar to tropical sprue.
• Histology: contain numerous refractile globules in membranes. Acid-fast staining variable.

Clinical Manifestations of Cyclosporiasis


• Symptoms: usually in a cyclical pattern alternating with constipation
1. may have flu-like prodrome with myalgias
2. watery diarrhea: relapsing, cyclical pattern
3. profound fatigue
4. anorexia
5. weight loss
6. less common: fever, chills, headache, vomiting
• self-limited infection in immunocompetent, but may last weeks
• abnormal D-xylose absorption in infected patients
• infection in AIDS patients is prolonged, severe and high recurrence rate >>> treat with long-term suppressive
therapy.
• Acquired immunity may not be completely protective

Diagnosis of Cyclosporiasis
• Microscopic demonstration of oocysts in fecal specimens, duodenal or jejunal aspirates or biopsy specimens.
• Identify acid-fast organisms (variable) with modified Ziehl-Neelsen or Kinyoun stains.
• Oocysts autofluoresce and appear as neon blue circles under UV fluorescent microscope.

Treatment of Cyclosporiasis
• TMP-SMX

Microsporidiasis
• Microsporidia are obligate eukaryotic intracellular parasites.
• Ubiquitous in environment, infect most animal phyla.
• Probably zoonotic and/or waterborne infections.
• Produces a wide array of clinical diseases in immunocompromised hosts.
• Over 1000 species in the phylum Microsporidia.
• Morphology: form very hardy characteristic unicellular ovioid spores w/ chitin & protein (1-3 microns in size).
A defining characteristic of all Microsporida is an extrusion apparatus consisting of a polar tube attached to inside
of anterior spore by an anchoring disk. During germination, the polar tube everts forming a hollow tube that
serves as a bridge to deliver sporoplasm to the host cell.
• Distribution: everywhere except Antarctica.
• Cause diarrhea.

Types of Microsporidia
• over 1,000 species in the phylum Microsporidia. Some important pathogens are:
1. Vittaforma corneae
2. Brachiola algerae
3. Pleistophora
4. Trachipleistophora: associated with encephalitis and disseminated disease.
5. Encephalitozoon hellem: associated with superficial keratoconjunctivitis, sinusitis, respiratory disease,
prostatic abscesses, and disseminated infections.
6. Encephalitozoon cunicili: associated with Hepatitis, encephalitis, and disseminated disease.
7. Encephalitozoon (Septata) intestinalis: associated with diarrhea, disseminated disease and superficial
keratoconjunctivitis.
8. Nosema, Vittaforma, and Microsporidium have been associated with stromal keratinis due to trauma in
immunocompetent.
9. Pleistophora, Brachiola, and Trachipleistophora have been associated with myositis.
10. Enterocytozoon bieneusi: associated with malabsorption, diarrhea, and cholangitis.

Pathogenesis of Microsporidia
• Common enteric pathogens that are usually self-limited and asymptomatic.
• Self-limited diarrhea in normal hosts.
• If immunocompromised: diarrhea, wasting syndrome, and disseminated infection.
• Enterocytozoon bieneusi most common, hard to treat, seen in liver and heart-lung transplant patients.
• Encephalitozoon species infect patients with kidney, pancreas, liver or bone marrow transplants.
• Prior to HAART, Microsporidia responsible for 1/3 of cases of chronic diarrhea in HIV+.

Clinical Manifestations of Microsporidia Infections


• GI Infections: infection of epithelium of GI tract is most common presentation, 90% due to Enterocytozoon
bieneusi, and less commonly due to Enephalitozoon intestinalis. Can lead to Sclerosing Chonlangitis,
Cholangiopathy, and Cholecystitis in AIDS patients (most patients have elevated liver function tests including
alkaline phosphatase, gamma-glutamyltransferase, aspartate aminotransferase and alanine aminotransferase).
1. E. bieneusi is found on apical surface of enterocytes of small intestine and epithelial cells of biliary tract
and pancreas. Rarely disseminates, unlike Encephalitozooindae, which are commonly found in lamina
propria and visceral organs. Symptoms: chronic diarrhea (3-10 loose watery stools per day), weight loss
and bloating, no fever. Most common in AIDS patients w/ CD4 <50. High mortality.
2. E. intestinalis is invasive and spores common on apical and basal sides of intestinal enterocytes and
inside fibroblasts, macs, and endothelial cells in the lamina propria.
• GU Infection: Encephalitozoon species infect GU of most mammals. Transmission through contamination of
environment with spores passed in urine. Infections can affect any organ (eyes, GI, liver, CNS, etc.). HIV
patients with keratitis can have asymptomatic infection of urinary tract and bronchial tree.
• CNS Infection: Granulomatous encephalitis is classic presentation of E. cuniculi; can mimic toxoplasmic
encephalitis in HIV patients.
• Ocular Infection: E. cuniculi, E. hellem & E. intestinalis can result in punctuate keratopathy and conjunctivitis
characterized by multiple punctuate corneal ulcers, e.g. superficial epithelial keratitis.
• Musculoskeletal Infection: myositis with inflammation due to several different Microsporidia has been described
in humans. This includes Pleisophora ronneafiei, Pleistophora species, Trachipleistophora hominis, Brachiola
vesicularum and Brachiola algerae.
• Sinus & Respiratory Infections: in disseminated infection due to Encephalitozoonidae, respiratory involvement
is often seen. The pathologic features are non-specific and may include rhinitis, sinusitis, and/or nasal polyposis in
any combo.

Diagnosis of Microsporidiosis
• NOT acid fast – use Trichome Stain!!
• Examination of stool specimens with light microscopy
• If stool is negative after 2+ months diarrhea, do endoscopy
• Obtain urine specimens b/c shedding of spores in urine is common in all species of Microsporidia.

Treatment of Microsporidiosis
• Microsporida that disseminate (like Encephalitozoon) are usually sensitive to Albendazole.
• E. bieneusi resistant and often treated with Fumagillin.
• Effective HAART >>> patients with improved immune function can usually fight off infection on their own.
LECTURES #12-13: MALARIA

Malaria
• Malaria is an acute and chronic disease caused by obligate intracellular protozoa of the genus Plasmodium.
• Four species of Plasmodium can infect humans:
1. P. malariae
2. P. vivax: present in Mexico, Central & South America, India, SE Asia
3. P. falciparum: associated with severe and complicated disease; predominant species in Africa,
Haiti, Dominican Republic; present in Mexico, Central & South America, Indian subcontinent, SE
Asia, and Oceania.
4. P. ovale: mainly in Africa
• Transmission: parasites are transferred to humans via Anopheles mosquitoes.
• Clinical Presentation: illness is highly variable, but characterized by fever, chills, anemia, and splenomegaly.

Epidemiology of Malaria
• 100+ countries in Asia, Africa, Latin America, Caribbean and Pacific Islands.
• 300-500 million cases each year, 100 million annually in SSA
• 1 million death per year – mostly in infants and young children

Life Cycle of Malaria


A. Initial Infection
• Sporozoites (infective stage) are injected along with saliva into subQ caps when female mosquito drains blood
>>> present in peripheral blood for 30 min
B. Exoerythrocytic Phase
• some sporozoites are killed by phagocytes, others enter liver parenchymal cells >>> multiply asexually there in a
process known as exoerythrocytic schizogony/merogony.
• 6-16 days later, hepatic cells with schizonts/meronts rupture >>> merozoites enter circulating RBCs.
1. P. falciparum and P. malariae schizonts rupture at about the same time, and none persist in liver. No
persistent exoerythrocytic phase, so relapse does not occur.
2. P. vivax and P. ovale have 2 types of exoerythrocytic forms:
a. primary type develops and ruptures in 6-9 days
b. secondary type (hypnozoite), may remain dormant in liver for weeks, months, or up to 5 years
before developing and causing relapse of infection (known as recrudescence).
C. Erythrocytic Stage
• Merozoites released from tissue schizonts invade RBCs (mediated by glycophorins on RBC surface) through
invagination at electron dense annular junction.
• Internal parasite organelles (rhoptries and micronemes) discharge their contents, and the merozoite moves into the
invagination.
• Once inside the cell, the parasite is sheathed in the parasitophorous vacuole membrane.
D. Asexual Stages
• The youngest stages in the blood are small, rounded trophozoites, known as ring forms.
• As parasites grow, they become more irregular and amoebid, and consume hemoglobin >>> producing iron-
containing byproduct heamtin or hemozoin, which is visible in cytoplasm of the parasite as dark granules.
• Erythrocytic schizogony/merogony: the schizont/meront stage begins when the parasite undergoes nuclear
division and culminates in segmentation to form merozoites.
• The erythrocytic cycle of schizogony continues destroying more RBCs until the host’s immunity or antimalarial
drugs stop it.
E. Sexual Stages
• The parastite life cycle continues when subpopulations of merozoites differentiate into sexual forms known as
gametocytes.
• Female macrogametocytes and male microgametocytes usually appear within 3-15 days of the onset of
symptoms.
The malaria parasite life cycle involves two hosts. During a blood meal, a malaria-infected female Anopheles mosquito
inoculates sporozoites into the human host . Sporozoites infect liver cells and mature into schizonts , which rupture
and release merozoites . (Of note, in P. vivax and P. ovale a dormant stage [hypnozoites] can persist in the liver and
cause relapses by invading the bloodstream weeks, or even years later.) After this initial replication in the liver (exo-
erythrocytic schizogony ), the parasites undergo asexual multiplication in the erythrocytes (erythrocytic schizogony ).
Merozoites infect red blood cells . The ring stage trophozoites mature into schizonts, which rupture releasing merozoites
. Some parasites differentiate into sexual erythrocytic stages (gametocytes) . Blood stage parasites are responsible for
the clinical manifestations of the disease.
The gametocytes, male (microgametocytes) and female (macrogametocytes), are ingested by an Anopheles mosquito
during a blood meal . The parasites’ multiplication in the mosquito is known as the sporogonic cycle . While in the
mosquito's stomach, the microgametes penetrate the macrogametes generating zygotes . The zygotes in turn become
motile and elongated (ookinetes) which invade the midgut wall of the mosquito where they develop into oocysts . The
oocysts grow, rupture, and release sporozoites , which make their way to the mosquito's salivary glands. Inoculation of
the sporozoites into a new human host perpetuates the malaria life cycle .

Clinical Manifestations of Malaria


• Only asexual stages are associated with symptoms.
• Clinical outcome is usually decided by degree of parasitemia. >1% parasitemia 1% mortality, >5% parasitemia
73% mortality (P. Falciparum). Usually don’t have mortality with other strains, unless elderly, atherosclerotic
patient with P. Vivax.
• Flu-like prodrome: headache, malaise, myalgias, fever
• Malaria paroxysms (corresponding to rupture of RBCs) usually develop within a few days of prodrome.
1. fever (as high as 105F)
2. rigors
3. severe headaches
4. abdominal pain
5. nausea
6. splenomegaly
7. hepatomegaly
8. blood tests show: anemia, thrombocytopenia, neutropenia, hyponatremia, and mildly elevated
transaminases
• When majority of parasites in RBCs undergo schizogony (maturation) at same time, paroxysms occur at 48 hour
intervals (the length of life cycle for all species of malaria except P. malariae which is 72 hours). This is rare in P.
falciparum >>> see irregular pattern of paroxysms.
• Simple Tertian: Vivax & Ovale – individual has fever every 48 hours or 3 days.
• Simple Quartan: Malarie – every 72 hours or every 4th day
• Malignant Tertian: fever less intermittent, fever just keeps going up and down, but have well days in between.
Falciparum.
• Uncomplicated Malaria: 0.1% mortality rate, absence of features of severe malaria.
1. In adults common presentation is malaise, headaches, myalgias, abdominal pain early on, and later
Triphasic paraoxysms (severe chills, fever, sweats, vomiting & watery diarrhea). Chills particularly
severe with Vivax malaria. Spleen very enlarged and soft – if you poke too hard, you can rupture it.
2. In children: cough, tachypnea, febrile convulsions (not cerebral malaria), in addition to adult symptoms.
• Severe Malaria: more common in first few years of life, and les frequent with increasing age. High level of
circulating proinflammatory cytokines (TNF-alpha, NO, IL-1, IL-6, IL-8, IL-10, IL-12). A life threatening
emergency with asexual parasitemia and 1+ complications:
1. impaired consciousness or coma (coma doesn’t necessarily indicate cerebral malaria)
2. pulmonary edema (including ARDS)
3. metabolic acidosis (usually lactic acidosis, major cause of death)
4. high parasitemia >5%
5. circulatory collapse, shock
6. Acute renal failure
7. Severe anemia
8. Hypoglycemia (<40 gm/dL, normal = 80-100 gm/dL)
9. spontaneous bleeding
10. hemoglobinuria
11. generalized convulsions = 2+ seizures in 24 hours
• lactic acidosis: major cause of death, anaerobic metabolism of parasites, decreased lactate clearance by liver,
reduced delivery of oxygen to tissues due to sequestration, resetting, and hemolysis.
• Pathogenesis of coma unclear: cerebral edema not significant, intracranial pressure increase is not a significant
factor.
• Thrombocytopenia with all 4 species of malaria due to increased splenic platelet clearance, platelet turnover will
increase w/ time, so not usually a serious complication. Etiology unclear.
• Hypoglycemia: young children and pregnant women, due to decreased oral intake, depletion of liver glycogen by
glycogenolysis, parasite consumption of glucose, increased insulin release from the pancreas (due to Quinine or
Quinidine), inhibition of glyconeogenesis (due to increased TNF-alpha and IL-1).
1. very serious complication in pregnancy! Fetus can have permanent CNS damage, IUGR, stillbirths or
spontaneous abortion. Malaraial parasites will bind to syncitiotrophoblast (but doesn’t happen after
second pregnancy).
• Nephrotic Syndrome fatal complication of malaria.

Immunity to Malaria
• Merozoite of P. vivax needs Duffy antigen on surface of RBC >>> lack of Duffy antigen 100% protective against
PV infection.
• In West Africa, 98% of local population lacks Duffy antigen.
• P. Ovale also found in West Africa, but can invade regardless of Duffy antigen.
• Sickle cell trait provides 90-95% protection from malaria, b/c HbAS clears organisms from spleen more quickly
and inhospitable pH. Spleen key site of antibody production >>> phagocytosis of merozoites, infected RBCs, and
debris, and Cytotoxic killing of infected RBCs.
• HbF in newborns provides protection from malaria for 5-6 months.
• After a long, unbroken period of heavy exposure, strain-specific clinical immunity can develop in adults, but not
permanent.
• Malaria immunity: TH1 regulated (TNF-alpha & IFN-gamma) initially >> TNF causes fever which inhibits
parasite growth. Later antibody depedent TH2 regulated >>> opsonization of parasites and infected RBCs with
IL-4, IL-5, IL-6, IL-10.

Complications of P. Falciparum Infection


• Life-threatening complications can develop from P. falciparum infection, because it can infect RBCs of any age,
unlike other species of Plasmodium (P. vivax and P. ovale infect Reticulocytes and P. malariae older erythrocytes).
• Also, PF-infected RBCs adhere to postcapillary venules and uninfected RBCs.
• Severe anemia, parasitemia and cytoadherence >>> tissue hypoxia and organ dysfunction >>> can lead to
1. symmetrical encephalopathy (known as cerebral malaria)
2. acute tubular necrosis
3. Adult Respiratory Distress Syndrome (ARDS)
• Clinical manifestations of severe PF disease include:
1. depressed consciousness
2. convulsions
3. respiratory distress
4. shock
5. spontaneous bleeding
6. jaundice
7. Acute Renal Failure
8. hematuria
• Lab values associated with severe PF disease:
1. >2% parasite density
2. schizonts in peripheral blood smear
3. acidosis
4. hypoglycemia
5. prolonged clotting time
6. thrombocytopenia
7. hemoglobin <7.5 g/dL

Cerebral Malaria
• Life-threatening complication of P. falciparum infection.
• Coma or impaired neural status during malaria infection needs to be distinguished from other causes of neuro
deficits such as hyperpyrexia, hypoglycemia, and concurrent infection.
• Seizures >>> coma or decreased consciousness.
• Nuccal rigidity and photophobia do NOT occur, but patients can develop neck retraction, opishotonos, gaze
disorders and posturing.
• Pathogenesis: altered RBC rheology, upregulation of cytokines (TNF-alpha, IL-1), upregulation of endothelial
receptors (ICAM-1, CD36, ELAM-1, Thrombosondin), cytoadhesion (high sequestration in brain capillaries
correlates with high mortality).
• Corticosteroids can increase mortality.
• Antipyretics, antiemetics, and anticonvulsants can be used.

Blackwater Fever
• Etiology unknown, but may be the following:
1. G6PD deficient individuals with ingestion of oxidant drugs +/- malaria.
2. G6PD deficient + malaria + Quinine
3. severe quinine-treated malaria with normal G6PD
4. unknown how Quinine is involved.
• Hemoglobinuria
• Jaundice
• Dark urine
• Oliguria or anuria (typically <200 ml/day)
• Nausea & vomiting of bile

Malaria During Pregnancy


• Can compromise fetal development and induce premature labor or spontaneous abortion.
• Chloroquine is safe for fetus.
• Concerns about oxytocic effects, but Quinine and Quinidine can be used safely during pregnancy. The risks of
malaria infection outweight concerns about drugs.
• Theoretical risk of hyperbilirubinemia and subsequent kernicterus with Sulfonamide use during pregnancy.
• Tetracycline contraindicated during pregnancy >>> treat malaria during pregnancy with Quinine and Clindamycin.
• Primaquine is also contraindicated during pregnancy >>> patients should receive Chloroquine alone and be
monitored for relapse.
• Congenital transmission can occur.

Malaria & HIV


• increased HIV replication and increased viral load
• greater parasitemia
• higher fevers, greater anemias, adverse birth outcomes (all pregnancies at risk rather than just first and/or second
pregnancies).

Diagnosis of Malaria
• Malaria is a lab diagnosis. Important to examine several sequential blood smears for presence of parasitized
erythrocytes. Initial smear may be negative, b/c symptoms can occur a few days before the appearance of infected
erythrocytes at detectable levels.
• Ideally, both thick & thin smears should be obtained and examined on multiple occasions over a 72-hour period
before diagnosis of malaria is excluded.
• New tests can detect parasites at lower levels.
• Rapid Diagnostic Tests (RDT), which are immunochromatographic, are based on capture of parasite antigen
from peripheral blood using monoclonal antibodies prepared against a malaria antigen target.
1. malaria antigens currently targeted by RDT are P. falciparum histidine-rich proteins
(HRP-2), pLDH, and Plasmodium aldolase.
• Thick blood smears are prepared with Giemsa stain in a manner that optimizes parasite detection, but is
unreliable for identification of species. Uses 30-40 microliters of blood >>> can’t see through it on microscope so
lyse RBCs and then stain.
• Thin blood smears are used for species identification, and prepared same as routine hematologic smears. Uses
about 10 microliters of blood, morphology of parasite in relation to RBC is preserved.
• In addition to species identification, the degree of parasitemia should be estimated >>> prognostic implications in
severe PF infections. At low levels of parasitemia, this can be done via thick smears, but at higher levels smears
should be examined at several oil immersion fields.
• Blood cultures should be obtained due to possible presence of coincidental bacterial infection (e.g. Typhoid).
• Should not have parasites in blood after 4-5 days of treatment >>> if there are, then organism is chloroquine
resistant.

Left image: Plasmodium falciparum, ring forms. Right image: Plasmodium falciparum diagnostic sausage or banana-
shaped gametocyte.
Initial Evaluation of Newly Diagnosed Case
• Percentage parasitemia
• Blood cell count
• Platelet count
• Electrolytes
• Creatinine levels
• Urea
• Ca2+
• Glucose
• Liver enzymes
• Coagulation tests
• In patients with severe malaria or symptoms of respiratory distress, lactate and blood gas analysis should be
obtained.
• After treatment or before travel, glucose 6-phosphate dehydrogenase levels should be determined.

Differential of Flu-like Symptoms in Traveler


• Malaria
• Typhoid Fever
• Early Meningococcal disease
• Rickettsiosis (Typhus)
• Arboviral infections like Dengue Fever
• Leptospirosis

Treatment for Malaria


• Antimalarials are generally more toxic than antibiotics, but few serious adverse effects ordinarily.
• 5 broad categories:
1. Quinolines: inhibit Heme Polymerase
• Quinine
• Quinidine
• Choloquine
• Mefloquine
• Primaquine
• Tafenoquine
2. Antifols:
Pyrimethamine
Proguanil
Trimethoprine
3. Artemesinin cpds:
Artemesinin
Artemether
Artesunate
4. Antibacterials:
Tetracylcines
Sulfonamide
Clindamycin
5. Atovaquone-proguanils: combo of hydroxnapthquinone + dihydrofolate reductase inhibitor
• Patients with P. falciparum infections, mixed infections with PF, and infections with unidentifiable strain should
be hospitalized.
• Patients with non-falciparum malaria should be given a course of Chloroquine phosphate (oral), followed by
Primaquine phosphate if PV or PO (after screening for G6PD Deficiency).
• Patients with P. falciparum malaria acquired in Central America west of Panama Canal, Mexico, or Middle East
can take Chloroquine – Falciparum malaria acquired anywhere else should be considered resistant.
• Chloroquine binds to heme with buildup of Chloroquine-heme complexes >>> irreversibly toxic. Resistance to
Chloroquine is all over the world.
1. resistance due to single mutation on PfCRT (Plasmodium falciparum Chloroquine Resistance
Transporter) which allows pumping out of Chloroquine from lysosomal vacuole >>> allowing
polymerization to occur >>> parasites are not eradicated.
• Mefloquine rare resistance in Asia and South America. Very effective, but major problem with side effects –
particularly in those with depression or anxiety disorders >>> use Tetracylcine or Malarone instead. Also, severe
nightmares.
• Quinine is drug of choice. For patients with CRPF, Quinine is best treatment for 3 days (or 7 days if acquired in
SE Asia or Oceania due to demonstrated reduced sensitivity to Quinine). Regimen should be supplemented with
Tetracyline, Clindamycin, or Pyrimethamine-Sulfadoxine to avoid recrudescence.
• Patients with decreased sensorium should be given IV Quinidine gluconate for 3 days with 5% glucose, and
Tetracycline or Clindamycin, and should be monitored.
• Severe anemia >>> transfusion
• Hypoglycemia common in pregnant women, children and severe cases
• In non-immune patients with Falciparum malaria, mortality >60% with parasitemia >10%. Exchange transfusion
or pheresis can be used to remove infected erythrocytes in cases of hyperparasitemia.
• Renal failure can occur secondary to dehydration. Peritoneal dialysis can be lifesaving for patients with ARF.
• Severe malaria can be complicated by non-cardiogenic pulmonary edema (ARDS) >>> poor prognosis.
• Artemisinin: very effective anti-malarial drug used in China since 371 AD, isolated from Sweet Wormwood
plant (Artemisia annua), very effective against PF & PV (but Mefloquine is more effective against PV). Pretty
cheap. Quickens parasite and fever clearance times, and shorter coma resolution than Quinine – works within 24
hours by acting on ring stages, whereas most other anti-malarials only work against schizont forms.
1. can be used orally, parenterally or as sepository
• Atovaquone-Proguanil (Malarone): used for uncomplicated malaria, b/c no IV or sepository forms.
Mechanism: inhibits parasite mitochondrial electron transport system at cytochrome bc1 and collapses
mitochondrial membrane potential >>> kills parasite. Proguanil is a prodrug that normally functions as DHFR,
but in this case it binds to Atovaquone and transports it across mitochondrial membrane.
• P. vivax and ovale need to use Chloroquine + primaquine (which kills liver stage), whereas with Falciparum &
Malariae need only Chloroquine.
• Severe malaria always parenteral drug – Quinidine or Quinine (not available in US). Quinidine is much more
cardiotoxic.
• Primaquine can be used for self-treatment of relapsing malaria.

Malaria Prophylaxis
• Doxycycline: problems is side effects like photosensitivity, Stevens-Johnson syndrome, and GI effects.
• Primaquine: good with minimal side effects, but not FDA approved yet.
• Malarone: most popular, few side effects, but very expensive.
• Mefloquine: very effective, but major problem with side effects – particularly in those with depression or anxiety
disorders >>> use Tetracylcine or Malarone instead. Also, severe nightmares.

Prevention of Malaria
• Sleep in screened rooms, and use permethrin-treated bed nets
• Pyrethrum insect spraysor repellents with N, N-diethyl-m-toluamide (exposure to high levels for prolonged
periods of time >>> neurotoxicity, particularly seizures in children, so use <35% DEET)
• Minimize exposed skin from dusk until dawn
• Use Mefloquine or Atovaquone-proguanil for prophylaxis (except in few areas where chloroquine resistant malaria
has not been described). Alternative: Doxycycline.

Anti-malarial Drugs
• Chloroquine phosphate (Aralen)
• Chloroquine sulfate (Nivaqine)
• Mefloquine (Larium)
• Quinine & Quinidine
• Primaquine phosphate
• Pyrimethamine-sulfadoxine (Fansidar)
• Tetracylcines: Tetracylcine, Doxycycline, Demeclocycline & Minocycline
• Macrolides: Erythromycin, Azithromycin, Clarithromycin
• Lincomycins: Clindamycin (Cleocin)
• Proguanil-Atovaquone (Malarone)
• Artemesinin (Artesunate & Artemether)
• Halofantrine (Halfan)
• Fluorquinolones

Chloroquine
• Chloroquine phosphate (Aralen) & Chloroquine sulfate (Nivaqine) are 4-aminoquinolones that act on the
erythrocytic stage of all species of Plasmodium.
• Drug of choice for treatment of susceptible strains of PF, PV, PO, and PM.
• Does not affect liver schizonts >>> doesn’t prevent relapse of P. vivax or P. ovale >>> subsequent treatment with
Primaquine should be used to avoid relapses.
• Primaquine should NOT be given to patients with G6PD deficiency.
• Adverse Effects: occur in 25%
1. GI upset
2. visual disturbances
3. headache
4. non-allergic pruritis
5. Via IV, hypotension and heart block can also occur, particularly if drug is administered rapidly.

Mefloquine
• Mefloquine/Larium is a r-quinolinemethanol that acts on the erythrocytic stage of all species of Plasmodium -
including most Chloroquine-resistant P. falciparum (CRPF). Does not affect liver schizonts.
• In US, recommended for prophylaxis not treatment, b/c effective treatment doses >>> CNS effects including
seizures and psychosis. Can be confused with Cerebral Malaria.
• Contraindicated in patients with history of:
1. epilepsy
2. psych disorders
3. cardiac conduction disorders (b/c can cause QT prolongation)
4. use with caution with patients on Beta blockers
• In theory, Quinine & Quinidine may exacerbate the cardiodepressant effects of Mefloquine >>> so loading doses
should not be given in patients who have recently taken Mefloquine.
• Teratogenic in rats, but has been taken in 2nd and 3rd trimesters w/o reported fetal malformations; 1st trimester
effects are unknown. Should be avoided in pregnancy.
• Due to long half life, patients should wait 3 months after stopping Mefloquine before getting pregnant.

Quinine & Qunidine


• Quinine Sulfate (oral) and Quinidine Gluconate (IV) are Cinchona Alkaloids.
• Very effective anti-malarials against all 4 species, also active against the gametocytes of PV, PO, and PM
• First-line treatment for Chloroquine resistant malaria >>> should be used as empiric treatment in all cases where
species is unknown:
• Quinine (650 mg 3x/day for 3-7 days)
1. Pyrimethamine-Sulfamethoxine, Tetracycline or Clindamycin (Cleocin)
2. For severe cases, IV Quinidine gluconate + Tetracylcine or Clindamycin
• Adverse Effects:
1. Cinchonism: syndrome caused by high serum levels of Quinine or Quinidine, includes headache,
nausea, abdominal pain, tinnitus, and transient loss of hearing; begins 48 hours after treatment is
initiated, stops soon after treatment ends.
2. hypoglycemia: give pregnant women IV glucose on these drugs
3. QRS widening and arrythmias >>> parenteral Quinidine should be done with EKG monitoring.

Primaquine Phosphate
• Is an 8-aminoquinolone active against tissue schizonts and gametocytes
• Indication: prevent relapse of PV or PO
• Contraindicated in patients with G6PD deficiency (causes hemolysis), and in pregnancy

Pyrimethamine-sulfadoxine (Fansidar)
• Indication: treatment of CRPF malaria
• Not used for prophylaxis due to fatalities from serious cutaneous allergic reactions (Stevens-Johnson Syndrome)
occurs in 1/20,00 patients with 5 weeks
• Patients with G6PD deficiency can develop hemolysis

Tetracyclines
• Types: Tetracycline, Doxycycline, Demeclocycline, Minocycline
• Indication: active against CRPF malaria; some activity against schizonts, but not enough to be used to prevent
relapse.
• Doxycycline (Vibramycin) is used for prophylaxis in patients unable to take Mefloquine
• or in areas of Mefloquine resistance (Vietnam and Thai-Kampuchean border).
• Adverse Effects:
1. GI intolerance
2. photosensitivity
3. vaginal candidiasis >>> women can treat with Fluconazole

Macrolides & Lincomycins


• Indication: active against erythrocytic schizonts of all species; Clindamycin + Quinine for treatment of CRFP.
Azithromycin has been used prophylactically in trials.
• Adverse Effects: Clostridium difficile infection >>> can be managed by Metronidazole.

Proguanil-Atovaquone (Malarone)
• Indication: active against erythrocytic and extraerythrocytic schizonts. Atovaquone is highly effective against PF,
but high recrudescence rate with development of resistance when used as monotherapy. So used in combo with
Proguanil.
• Mechanism: inhibits mitochondrial electron transport.
• Adverse Effects: abdominal pain, diarrhea, nausea, coughing.

Artemisinin
• Derived from Chinese medicinal plant Qinghao (Artemisia annua)
• Rapidly acting anti-malarial effective against multidrug resistant P. falciparum malaria.
• Mode of delivery: IV, oral, suppository
• Not available in US

Halofantrine (Halfan)
• Is a phethrenemethanol derivative of an amine alcohol that requires fat for optimal absorption.
• Recrudescence is a reported problem after single dose treatment.
• Adverse effects: cough, pruritis, hemolytic anemia, QT prolongation, PR prolongation, Torsade de pointes. Use
limited by side effects and cardiotoxicity.

Fluoroquinolones
• Have anti-malarial activity in vitro, but not proven effective in vivo.

Babesiosis
• Distribution: localized to only NE US (particularly Cape Cod, Martha’s Vineyard, CT, & Long Island)
• Disease due to Babesia microti, which leads to infection of RBCs in wild and domestic animals with a malaria-
like organism.
• Transmitted by Ixodes ticks. A 3 host tick: hatches from egg >>> becomes larvae >>> feeds on animals such as
mice and birds >>> molts into nymph stage >>> becomes active the following year >>> looks for another host to
feed again >>> adult female feeds on deer and that’s where males and females copulate. Deer is not a host – just
site of mating and food.
• Hemolytic anemia, fever, hemoglobuinuria and malaria-like systems.
• Particularly problematic in people without spleens!
• Can have co-infection with AIDS, Ehrlichiosis & Lyme Disease.
• Diagnosiss: blood smear and PCR, look for Maltese Cross on blood smear – pathognomonic!
• Treatment: Clindamycin or Erythromycin + Quinine. Atovaquone + Azithromycin best treatement, particularly
in AIDS patients.
Image of Babesisos: with Maltese Cross!

LECTURE #14: AFRICAN & AMERICAN TRYPANOSOMIASIS

Hemoflagellates/Kinetoplasts
• Have a single large organelle that is basically a mitochondria. Called a hemoflagellate because found in blood.
1. American Trypanosomiasis/Chagas’ Disease: Trypanosoma cruzi, vector is Reduvid bug
2. African Tropanosomiasis/African Sleeping Sickness Trypanosoma: Trypanosoma brucei gambiense,
vector is Tsetse fly
3. African Tropanosomiasis/African Sleeping Sickness: Trpanosoma brucei rhodesiense, vector Tsetse fly
4. Animal trypanosomiasis in Africa
5. Leishmania: cutaneous & visceral (vector is Sandfly)

American Trypanosomiasis or Chagas’ Disease


• Caused by protozoan Trypanosoma Cruzi. Vectors often called kissing bugs.
• Host response is just as important as treatment – original case (Bernice) resolved spontaneously.
• Cause pseudocysts filled with intracellular forms: round with nucleus and cigar-shaped body which is the
kinteoplast. Amastigote – has no flagella.
• Distribution: from Mexico (or Southern US according to CDC) down into South America through Argentina.
Southern Cone Initiative: a bunch of South American countries have gotten together to eradicate. Worst areas
are:
1. Panama
2. Bolivia
3. southern Mexico
• Vector: infected triatomine insect vector (or “kissing” bug)
• Romana’s sign is a type of Chagoma (indurated erythematous lesion at site of entry) seen in acute phase of
disease, refers to unilateral periorbital edema just after bug bite.
• Every nucleated cell in body can be infected by this organism.

Morphology of Trypanosoma Cruzi


• Large posterior staining kinteoplast
• Flagella is anterior
Above image: Stages of Trypanosoma cruzi: amastigote (intracellular vertebrate), epimastigote (invertebrate),
trypomastigote (vertebrate, blood); metacyclic trypomastigote in insect rectum (not shown).

Transmission of Chagas’ Disease


• Normal transmission: bug bite, “kissing” bug bites at night (often lips or around eyes) typically kids.
• Other modes of transmission: blood transfusion, maternal milk, organ transplantation, lab accident, congenital,
oral transmission through contaminated sugar cane

Life Cycle of Trypanosoma Cruzi

Life Cycle:
An infected triatomine insect vector (or “kissing” bug) takes a blood meal and releases trypomastigotes in its feces near
the site of the bite wound (Chagoma forms around lips or eyes of child that was bitten). Trypomastigotes enter the host
through the wound or through intact mucosal membranes, such as the conjunctiva . Common triatomine vector species
for trypanosomiasis belong to the genera Triatoma, Rhodinius, and Panstrongylus. Inside the host, the trypomastigotes
invade cells, where they differentiate into intracellular amastigotes . The amastigotes multiply by binary fission and
differentiate into trypomastigotes, and then are released into the circulation as bloodstream trypomastigotes .
Trypomastigotes infect cells from a variety of tissues and transform into intracellular amastigotes in new infection sites.
Clinical manifestations can result from this infective cycle. The bloodstream trypomastigotes do not replicate (different
from the African trypanosomes). Replication resumes only when the parasites enter another cell or are ingested by another
vector. The “kissing” bug becomes infected by feeding on human or animal blood that contains circulating parasites .
The ingested trypomastigotes transform into epimastigotes in the vector’s midgut . The parasites multiply and
differentiate in the midgut and differentiate into infective metacyclic trypomastigotes in the hindgut .

Trypanosoma cruzi, amastigotes, in cell culture.


Clinical Presentation of Chagas’ Disease
• Chagoma, indurated erythematous lesion at site of Reduvid bite 10-14 days later.
• Diagnosis often missed. In endemic areas up to 80% seropositive, but don’t even remember being sick due to
spontaneous resolution.
• Acute disease: most asymptomatic (can remain asymptomatic for 10-20 years), some have fever, malaise,
sweating, myalgias, anorexia, lymphadenopathy, edema, hepatomegaly.
• Most patients asymptomatic, or:
1. Chronic Phase
2. GI: megacolon, megaesophagus, mega-gall bladder >>> any organ that is innervated by ANS
with tubular cells can be affected.
3. Cardiac/Chronic cardiomyopathy: DCM, CHF, Apical aneurysm (almost pathognomonic for
Chagas!), Thrombo-embolic events, Electrophysiological abnormalities.
• Common EKG findings:
1. RBBB
2. RBBB + anterior fasicular block
• Right side of heart always more involved than left side.
• Kids that are very infected can die of meningoencephalitis or cardiac problems (arrhythmias, acute heart failure),
and vasculitis.
• Prognosis: 2-5% mortality in kids, typically due to heart failure, seizures, or secondary infection. Most people go
on to indeterminate period for 10-20 years, followed by onset of chronic symptoms.
• Every component of Immune System important in fighting this infection: ABs, T cells, B cells. As antibodies
rise, parasitemia wanes, but have disruption of ganglionic cells due to surrounding inflammation (so any tubular
structure in the body innervated by ANS can be affected!) >>> as patient goes into chronic phase, don’t see
parasites in blood, but see evidence of infection through PCR, and if patient is immunosuppressed get exacerbation
of illness >>> so we know parasite is still there, but cant’ see it. Important b/c patient will get worse with
corticosteroids, HIV, or if patient donates blood, will transmit disease even though undetectable in blood.

Diagnosis of Chagas’ Disease


• During acute phase can see Trypanomastigotes in blood
• Xenodiagnosis: take a lab Reduvid bug, have them feed on patient, 35-40 days later look at rectum of bug for
parasites.
• Biopsy of various organs
• Serological tests for antibody – done here at Einstein
• Detection of parasite antigen

Left image: Trypanosoma cruzi in blood smear; note "C" shape and large posterior kinetoplast. Right image:
Xenodiagnosis of Chagas' disease with Reduviid bug.

Specific Problems
• Transfusion Chagas’ Disease
• HIV/AIDS: acts as OI in AIDS patients, have necrotizing encephalitis and acute myocarditis.
• Congenital Chagas’ Disease
• Transplantation (organ and stem cell)

Treatment of Chagas’ Disease


• Nifurtimox & Benznidazole: need to get from CDC. Both very toxic so add Interferon gamma (to arm
macrophages to better kill intracellular parasites).
• Treat patients with acute disease, but not necessarily chronic cases (international panel recommends treatment for
all cases).
• Other experimental drugs: Allopurinol, Antifungal Azoles

Prevention of Chagas’
• Avoid sleeping in dilapidated dwellings
• Insect repllents
• Mosquito nets
• Avoid blood transfusions in Latin America

African Sleeping Sickness


• Trypanosoma brucei rhodesians & gambiense are kenteoplasts.
• Distribution: endemic to sub-equatorial Africa
• Transmission:
1. via Glossnia or Tsetse Fly
2. direct mechanical
3. congenital
4. lab accident
• Trypanosoma brucei gambiense/African Sleeping Sickness (Gambian or West African form): evolves more
slowly, over many years, is fatal untreated; human-fly-human cycle. Found in West and Central Africa. Manifests
late, and can last for months to years. Diagnosis via node aspirate.
1. Glossnia palpalis
• Trpanosoma brucei rhodesiense/African Sleeping Sickness (Rhodesian or East African form): kills host
within weeks or months; maintained in wild animals, unless epidemics of disease, then human-fly-human. Found
in East and Central Africa. Manifests early w/ more anemia, shock, myocarditis prior to death. Higher level of
parasitemia. Diagnosis through blood.
1. Glosssnia morsitans
• Unlike related species T. cruzi & Leishmania there are no intracellular forms.
• While morphologically identical, have different biological properties, b/c humans only important host for TBG,
whereas animals infected by TBR.
• More likely to infect young men.
• At various stages, Trypomastigotes can be found in peripheral blood, lymphatics, lymph nodes, CSF, and neural
tissue.

Morphology of T. brucei & T. Rhodesiense


• Two subspecies are morphologically identical
• They are both kenteoplasts
• Pleomorphic flagellates 15-30 microns x 1.5-3.5 microns
• Long, slender w/ undulating membrane and free anterior flagellum, or short, broad w/o free flagellum

Life Cycle of Trypanosomes


• Blood forms/Trypomastigotes are ingested by Tsetse fly >>> posterior midgut multiply by binary fission for 7-
10 days >>> anterior foregut for 2-3 weeks >>> salivary glands replicate further >>> become infective metacyclic
trypomastigote forms >>> fly bites human >>> trypomastigotes multiply by binary fission in human blood,
lymph and extracellular space >>> invades CNS and continues to multiply.
• African Trypanosomiasis does NOT have an Amastigote phase (unlike American Trypanosomiasis)!

Life Cycle:
During a blood meal on the mammalian host, an infected tsetse fly (genus Glossina) injects metacyclic trypomastigotes into
skin tissue. The parasites enter the lymphatic system and pass into the bloodstream . Inside the host, they transform into
bloodstream trypomastigotes , are carried to other sites throughout the body, reach other blood fluids (e.g., lymph, spinal
fluid), and continue the replication by binary fission . The entire life cycle of African Trypanosomes is represented by
extracellular stages. The tsetse fly becomes infected with bloodstream trypomastigotes when taking a blood meal on an
infected mammalian host ( , ). In the fly’s midgut, the parasites transform into procyclic trypomastigotes, multiply by
binary fission , leave the midgut, and transform into epimastigotes . The epimastigotes reach the fly’s salivary glands
and continue multiplication by binary fission . The cycle in the fly takes approximately 3 weeks. Humans are the main
reservoir for Trypanosoma brucei gambiense, but this species can also be found in animals. Wild game animals are the
main reservoir of T. b. rhodesiense.

Genetics of African Trypanosomes


• T. brucei contains 1000 genes capable of coding for variant surface glycoproteins (VSG), which are switched at
a rate of 10^-2 to 10^-6 switches per generation >>> main mechanism of evasion of host immune responses.
• Only 1 VSG site is active at any given time.
• Expression sites (ES) occur 15-20 times in genome, all at subtelomeric locations. 3 types of DNA
rearrangements are associated with ES switching:
duplicative transposition
telomere exchange
telomere conversion
• In addition to VSG genes, several upstream genes called expression site-associated genes (ESAGs) are also
transcribed.
• Another feature of trypanosome gene is the phenomenon of mitochondrial RNA editing. The kinetoplast DNA is
organized as interlocking supercoiled network of approximately 50 maxicircle molecules and few thousand
minicircle DNA molecules. They encode small guide RNAs that serve as templates for insertion or deletion of
Uridines in the primary RNA transcript. The maxicircle DNAs encode for a dozen different mitochondrial
proteins.
Pathogenesis of Trypanosomiasis
• Blood forms increase exponentially over next 2 weeks >>> then disappears from bloodstream >>> only to return
later on.
• Antigenic Variation: interval between waves of parasitemia is 1-8 days, with clinical symptoms accompanying
each bout. Each new wave has a new antigenic variant (via VSG), that can evade host immune response.
• B Lymphocyte Proliferation:
1. lymphadenopathy
2. immune complexes
3. anemia
4. secondary infections dues to bacterial pneumonia or meningitis, or malaria
• Marked early IgM response >>> polyclonal B cell activation >>> production of wide array of antibodies
including:
1. brain-specific autoantibodies
2. ABs to myelin basic protein
3. ABs to ganliosides & cerebrosides
• Also, circulating immune complexes
• Cell mediated immunity, NO may be important in depression of T Cell responsiveness and generalized
immunodepression.
• Main pathological lesions involve:
1. CNS
2. Posterior Cervical, Submaxillary, Supraclavicular, & Mesenteric Lymph Nodes
• Lymphatic hyperplasia w/ diffuse proliferation of lymphocytes, initial hemorrhage w/ large numbers of
trypomastigotes >>> later nodes small and fibrotic.
• Progressive, chronic leptomeningitis w/ Morula cells
• Organism found in brain tissue & CSF, and lymphocytosis in CSF
• Can see: Glomerulonephritis, Myocarditis, Pericardial Effusion, Pulmonary Edema, and Hypoplastic bone
marrow.

Clinical Manifestations of Trypanosomiasis


• Rhodesian infection is more fulminant and acute (weeks – 9 months), whereas Gambian infection can last for
years.
• Trypanosomal chancre within 2-3 days at site of bite.
• Large, red, painful nodule in 1 week (7-10 cm), lasts 2 weeks
• Symptoms: intermittent fevers, chills, headaches, back pain, generalized lymphadenopathy, moderate
hepatosplenomegaly, delayed deep hyperesthesia over tibia
• In Caucasians, can have erythema multiforme.
• Chronic, untreated Gambian disease: severe headaches, inability to sleep, progressive mental degeneration,
tremors, psych manifestations (depression, suicidal tendencies, schizophrenia) >>> coma >>> death.
• In Gambian form: when nodes in posterior cervical triangle are enlarged, it is known as Winterbottom’s Sign.

Lab Findings of African Sleeping Sicknes


• Anemia
• Thrombocytopenia
• Elevated ESR
• Elevated IgM in serum/CSF
• Trypanomastigotes in blood, CSF, LN

Diagnosis of Trypanosomiasis
• Serology and PCR – must use thick & thin smears and examine buffy coat!
Trypanosoma brucei rhodesiense in blood

Differential Diagnosis of Trypanosomiasis


• Malaria
• Typhoid Fever
• TB
• HIV

Treatment of African Trypanosomiasis


• Hemolytic stages or early stages:
Suramin: does not cross BBB
Eflornithine (DFMO)
Pentamidine
• Late stage with CNS involvement
1. Melarsoprol (Arsobal/Mel B): an arsenic compound that crosses the BBB and can cause reactive
arsenic-induced encephalopathy (RAS).
2. Eflornithine (DFMO): inhibits ODC, the lead enzyme in polyamine pathway.
• Relapse can occur (indicated by continued high IgM levels or abrupt rise in IgM).

Pentamidine
• 10 IM injections given on alternate days.
• Effective against early stage TBG, less so for TBR, and ineffective against later stage of both organisms.
• Does not kill organisms outright.

Diminazene Aceturate (Berenil)


• Low side effects
• Significant therapeutic value.
• Effective against early stage TBR & TBG, and used in combo w/ Melarsoprol for late stage disease.

Suramin
• Early stage TBR
• Given in 5 IV injections
• Very long half life
• Binds to plasma proteins including LDL which trypanosomes avidly bind to and endocytose >>> slow decrease in
parasite numbers.

Melarsoprol (Mel B or Arsobal)


• Can use to cure late stage CNS disease from TBR & TBG
• Series of 3 daily injections.
• Given IV.
• Mechanism: unclear.
• Toxicity: reactive arsenical-induced encephalopathy (RAS) >>> pulmonary edema & death in >50% cases within
48 hours. Occurs in 10% of patients. Co-administration of steroids can help.

Eflornithine (DFMO)
• Enzyme-activated inhibitor of ornithine decarboxylase (ODC), the lead enzyme of polyamine biosynthesis.
• Give IV 4 doses every 6 hours for 2 weeks.
• Short half life
• Costly and lengthy treatment
• Not trypanocidal, but blocks bloodstream division of trypanosomes.

LECTURE #15: LEISHMANIASIS

Types of Leishmaniasis
• Visceral Leishmaniasis (VL or Kala-Azar): fever, wasting, panctyopenia, HS, hyper-IgG
• Cutaneous Leishmaniasis (CL): skin ulcers
• Mucosal Leishmaniasis (ML or MCL): nasopharyngeal ulcers

Importance of Leishmaniasis
• Paradigm for studying cell mediated immunity, so important for scientific research.

Leishmaniasis
• Leishmania are hemoflagellates with a promastigote stage in insect vector and amastigote stage in
macrophages of vertebrate host (humans).
• Leishmaniasis is a disease caused by protozoa of the Leishmania species, which is transmitted by the bite of a
female sandfly. Clinically, Leishmaniasis is classified as:
Cutaneous Leishmaniasis (CL): skin ulcers
Mucocutaneous Leishmaniasis (ML or MCL): naso-pharyngeal ulcers.
Visceral Leishmaniasis (VL): fever, wasting, pancytopoenia, HS, hyper-IgG
• The prevalence of the disease is increasing, with estimates of 12 million people currently infected worldwide.
Once thought of as a disease affecting only rural areas, the disease remains common in the face of increasing
urbanization.
• Distribution: incidence is highest in tropical and subtropical regions where conditions are favorable for sandflies.
Leishmaniasis is found in some parts of 88 countries within Central America, South America, Africa, India, the
Middle East, Asia, southern Europe, and the Mediterranean.
Most Cutaneous leishmaniasis seen in Columbia, Brazil, Peru, Afghanistan, Iran, Iraq & Saudi Arabia
Visceral leishmaniasis is most common in Bangladesh, Brazil (epidemic in northeastern Brazil), India, Nepal,
and Sudan (on-going epidemic since 1992).
• Transmission: sandfly bite.
• Of great importance is the social burden placed by the morbidity of the disease, which can cause severe
deformities and disfigurment and lead to social isolation.
• Males are more commonly infected than females, with Visceral leishmaniasis in particular, having been shown to
be twice as common in males.
• Opportunistic infection in AIDS patients.

Life Cycle of Leishmania Species


• Sandfly bites an infected animal (like a dog) >>> ingests blood infected with Leishmania >>> bites human and
injects organism into skin >>> human is infected >>> another sandfly bites human and continues the cycle.
• The amastigote is an intracellular parasite that is ingested by fly >>> transforms into promastigote >>> colonizes
the hindgut of fly >>> colonizes midgut >>> migrates anteriorly into foregut >>> then goes to fly’s biting
apparatus >>> transforms into metacyclic Amastigote form (change in surface membrane of promastigote that
keeps it from being killed by complement in animal or human) >>> is injected into human.
• Gp63 & LPG on surface of organism bind complement and block membrane attack cascade of C5-C9 to avoid
killing, and blocks reactive oxygen intermediates in macrophage.
• Deposits eggs and then dies within 2-3 months. Lifespan of adult female is 2-3 weeks.
• Environments vary: some sandflies are desert dwellers, but most like moist, damp environments such as forests.
• Maxadilan: is a polypeptide in sandfly saliva extremely potent vasodilator (7x more potent than Nitroglycerin).
• L. chagasi causes Visceral Leishmaniasis in Brazil >>> has lots of maxadilan >>> can get into bloodstream and
go to various tissues all over the body. Whereas same organism in Costa Rica has lower maxadilan content
leading to Cutaneous Leishmaniasis. So sandfly has marked influence on what kind of disease individual will
present with.
• Flies also have important anti-clotting agent that keeps blood from clotting so that fly can feed on more blood in
animal/human tissue.

Causal Agent:
Leishmaniasis is a vector-borne disease that is transmitted by sandflies and caused by obligate intracellular protozoa of the
genus Leishmania. Human infection is caused by about 21 of 30 species that infect mammals. These include the L.
donovani complex with 3 species (L. donovani, L. infantum, and L. chagasi); the L. mexicana complex with 3 main species
(L. mexicana, L. amazonensis, and L. venezuelensis); L. tropica; L. major; L. aethiopica; and the subgenus Viannia with 4
main species (L. (V.) braziliensis, L. (V.) guyanensis, L. (V.) panamensis, and L. (V.) peruviana). The different species are
morphologically indistinguishable, but they can be differentiated by isoenzyme analysis, molecular methods, or monoclonal
antibodies.

Leishmaniasis is transmitted by the bite of female phlebotomine sandflies. The sandflies inject the infective stage,
promastigotes, during blood meals . Promastigotes that reach the puncture wound are phagocytized by macrophages
and transform into amastigotes . Amastigotes multiply in infected cells and affect different tissues, depending in part on
the Leishmania species . This originates the clinical manifestations of leishmaniasis. Sandflies become infected during
blood meals on an infected host when they ingest macrophages infected with amastigotes ( , ). In the sandfly's midgut,
the parasites differentiate into promastigotes , which multiply and migrate to the proboscis .

Identificaiton & Classification of Leishmania Species


• Restriction endonuclease digestion of Kinetoplast DNA
• Isozyme patterns specific to species
• DNA-DNA hybridization techniques
• Site of growth of promastigote in sandfly
• Growth in culture
• Species specific Monoclonal antibodies
• PCR species specific primers

Immune Response to Leishmaniasis


• Protective immune response is CMI, no role for antibodies in fighting this disease >>> so, there is a spectrum of
disease.
• VL & Diffuse Cutaneous Leishmaniasis: no CMI (no Delayed Hypersensitivity reaction), lots of parasites, lots of
macrophages, few lymphocytes, no Lymphocyte Blast transformation >>> Montenegro skin test is negative.
• Cutaneous Leishmaniasis: fewer parasites, fewer macrophages, some lymphocyte blast transformation >>>
Montenegro skin test is positive.
• Tuberculoid/ Leishmaniasis Recidivans: few parasites, few macs, LOTS of lymphocytes, lots of DH, lots of
lymphocyte transformation >>> very positive Montenegro skin test.
• IL-4, IL-10 & TH2 confers susceptibility. IL-10 associated with disease progression in humans. IL-12 promotes
TH-1 response. If person has good TH1 response, patient heals rapidly; if not, patient gets very sick!

Causative Organisms of Visceral Leishmaniasis


• L. donovani: India and East Africa
• L. infantum: southern Europe, northern Africa, Middle East, China
• L. chagasi: Western Hemisphere (probably same organism as L. infantum)
• L. tropica: old world
• L. amazonensis: new world in Central American and northern South America.
• Post-kala-azar dermal Leishmaniasis (PKDL): recurrence due to improper treatment

Epidemiology of Leishmaniasis
• Canine: Dogs in South America, Mediterranean, China, Brazil; Dogs >>> Foxes in Brazil; Central Asia: Jackals
• Rodents in East Africa (rats, gerbels, squirrels)
• India no reservoir; human hosts
• Congential, lab, person-to-person
• Immunosuppression: co-factor in HIV/AIDS; steroids; chemo; transplants

Visceral Leishmaniasis
• Kala-azar
• Mild visceral disease which does not progress
• Visceral disease as a component of AIDS
• Sudanese disease “killing disease” is very virulent + PKDL
severe burning feet
gall bladder disease (Cholecystitis)
congenital transmission common
PKDL in 56% of cases within 56 days.

Clinical Presentation of VL
• Onset: gradual to abrupt
• Incubation: 3-8 months (can be years)
• Fever: double daily spikes that can be variable or intermittent, can be misdiagnosed as Malaria
• Anemia: very characteristic of this disease b/c RBCs are hemolysed, sequestration of RBCs by Spleen, hemorrhage,
and marrow replacement.
• Leukopenia: Eosinophils are almost always absent, and low number of neutrophils.
• Low Platelets >>> bleeding.
• Hyperglobulinemia: CIC, RF, GN
• Hepatosplenomegaly: spleen usually enlarges before liver, and Splenomegaly is almost always seen with this disease.
• sometimes Lymphadenopathy
• Grayish or blackish skin: in African form usually hyperpigmented skin, whereas in Indian form skin is
hypopigmented.
• VL, DCL & Lepromatous Leprosy all can look the same clinically >>> biopsy to differentiate.

Diagnosis of Leishmaniasis
• Kinetoplast makes diagnosis – have to see this, otherwise could be confused with Chagas, Toxoplasmosis or other
diseases!
• Formalin Gel Test: not specific diagnostic test, but used frequently in field in countries where this disease is
common.
• Demonstration of organism on Giemsa stained slides with bone marrow biopsy, splenic aspiration (diagnostic
procedure of choice but highest risk), liver biopsy, buffy coat in blood (sometimes). Culture specimen in NNN
medium. Do isozyme analysis, use species specific DNA probe and PCR, or Monoclonal antibodies.
• Serological Diagnosis
use antibodies with IFA, ELISA, DAT. Don’t see ABs in CL >>> so negative, but will be positive in VL.
DHR: will be positive with CL, and negative with VL.

Treatment of Visceral Leishmaniasis


• Pentavalent Antimony (Sb+5): is gold standard for treatment, usually effective unless patient has resistant disease.
Parenteral. Cardiotoxic & toxic to pancreas and other organs.
• Amphotericin B: highly effective, used with patients who fail treatment on PA. Used in India, b/c high resistance to
PA, but has renal toxicities and is expensive. Parenteral.
• Lipid formulatios of Ampho B (Ambisome): more expensive, lower toxicities, and best treatment for AIDS patients.
Parenteral.
• Miltefosine: highly active oral agent, but not FDA approved yet, least side effects.

Prevention of Visceral Leishmaniasis


• No immuno or chemoprophylaxis
• Vaccines not available
• Vector control
• Elimination of reservoir hosts
• Avoid sandflies, use DEET, and sleep with Permethrin impregnated bed nets

VL Association with HIV


• Co-infection with HIV reported in 31 countries
• Particularly common in Mediterranean
• See VL when CD4 count drops to <50 (sometimes <200).
• See a lot of involvement of rectum, large intestine, pulmonary tissues (none of which are common in VL w/o AIDS).
• Poor response to Antimony. Almost 100% response to Amphotericin, but relapse soon after treatment is stopped.
Most effective treatment is probably using antiretrovirals to treat AIDS and bring CD4 count back up.

Distribution & Strains of Cutaneous Leishmaniasis


• Afghanistan (lots of cases), Brazil, Iran, Iraq, Saudi Arabia, Sudan & Syria
• Old World
L. tropica: urban disease, maintained in dogs & humans
L. major: rural disease, maintained by desert rodents, gerbils, rats
L. aethiopica: rock hyrax in Africa
L. infantum & L. donovani
• New World
L. mexicana: sp complex
L. braziliensis: sp complex
• Visceral disease is reported in Iraq, Iran & Saudi Arabia: visceral disease can be caused by L. tropica. LT can cause
both CL and VL – currently being seen in soldiers coming back to US from Iraq.
Pathogenesis of Cutaneous Leishmaniasis
• Fly bites >>> in a few weeks itchy papule or nodule develops (1 cm) >>> if enough IFN-gamma and CMI then nodule
heals, but if not, weeks to months later w/o CMI an ulcer will develop (up to a few centimeters in diameter) >>> in
months with CMI L. major will heal, but L. tropica may take years to heal.
• LT tends to visceralize, so common practice of self-innoculation has been discontinued.

American CL
• Zoonosis
• L. braziliensis: rodents, can metastasize to mucous membranes
• L. panamensis: sloths
• L. guyanensis: sloths and anteaters
• L. peruviana: dogs
• L. mexicana: rodents
• L. amazonensis: rodents and marsupials

Diagnosis of Cutaneous Leishmaniasis


• Demonstration of organism on biopsy
• Culturing with NNN
• Isozyme analysis, or species specific DNA probes
• Serological Diagnosis: through IFA, ELISA, DAT – low or absent in CL; high in VL. DHR is positive in CL after a
few months.

Treatment of Cutaneous Leishmaniasis


• Pentavalent antimonials: sodium stibogluconate & meglumine antimoniate
• Amphotericin B: very effective
• Pentamidine
• Paromycin
• Heat cuff

Das könnte Ihnen auch gefallen